Sie sind auf Seite 1von 263

Retail Bank

(McKinsey Quick on your Feet Competition)


Guidance for interviewer and
Problem statement narrative
information provided upon request
Our client is a bank in the US. It has a large retail
footprint and offers a mix of services to end-  Ensure the candidate is familiar with how a bank
customers (checking, debit, credit cards). They also earns its profits (e.g. the spread between what
have loan centers to sell mortgages in the same they lend money out at and borrowing costs, fees
markets. The bank currently serves 15 million on various services).
customers. Our client has historically been
profitable, but increased regulation and the downturn  Note that this is an interviewer-led case; there
in the economy have caused the bank to see a sharp are seven questions that the Interviewer will ask
decline in profitability. The client has engaged us to the Interviewee.
help determine next steps for its business and has
asked us to assess ways they can increase profits
within the next 12 months.

Difficulty: Medium Quant Heavy Industry: Financial Services Type: Profitability

- 27 -
Retail Bank
(McKinsey Quick on your Feet Competition)

Additional Questions to Steer Discussion


Questions for the candidate

1. What are some of the ways the bank can increase profits in the next 12 months?

A good answer is structured and contains a comprehensive set of ideas to both reduce costs and boost revenues as
well as clear examples. Interviewee should use a Profitability framework to approach this problem. Answers include
but are not limited to:
Reduce Costs
•Fixed Costs
•Reduce underperforming branches (close branches, lease branches to other banks)
•Reduce workforce (e.g., push greater use of online channels for banking, outsource functions)
•Consolidate the branches and the loan centers
•Variable
•Reduce costs associated with transactions (paper free, decrease error rate)
Increase Revenue
•Quantity
•Current Customers
•Cross-sell different products (home purchase mortgages, refinancing, credit card, debit card, money market,
advisory services)
•Change product mix to higher revenue products
•Get rid of unprofitable customers
•New Customers
• Increase Number of customers
•Product Mix
•Launch new products
•Price
• Increase bank fees (Debit fees, ATM fees, call center fees)
•Raise rates charged

- 28 -
Retail Bank
(McKinsey Quick on your Feet Competition)

Additional Questions to Steer Discussion


Questions for the candidate

We have worked with our client to narrow down their options to two choices. The first is shutting
down unprofitable retail locations, the second is a better customer segmentation strategy. Lets
explore both:

2. What are some of the risks with shutting down branch locations?

Note: there are a number of ways to think this through – look for the structure in how the candidate
responds. A good answer includes, but is not limited to:

Near Term
•Poor PR
•Legal/contractual complications
•Extra costs (severance)
•Lose a portion of customers who bank through that branch
•Selling off assets could scare investors

Long Term
•What happens when the market rebounds?

- 29 -
Retail Bank
(McKinsey Quick on your Feet Competition)

Additional Questions to Steer Discussion


Questions for the candidate

3. The second option the bank is considering is a better retail segmentation strategy: What
segments do you think a retail bank has?

Note 1: This response could have a lot of answers. Look for clear delineation between customers
who are profitable and unprofitable and then list characteristics of each (i.e. Segment 1 is mass
affluent and is highly profitable, uses checking accounts, has a money market account, and has a
mortgage with the bank; they are low cost as they generally use ATMs and the internet to manage
their transactions; Segment 2 is lower income, keeps a small balance in checking, uses tellers and
call centers often) etc.

Note 2: Ensure the candidate does not spend too long on this question.

4. Can you take a look at the below chart and walk me through what the bank is experiencing?
Please walk the interviewee through any questions they have on the chart in Exhibit 1:

Key insights include, but are not limited to:


- Only 30% of the bank’s customers are currently profitable
- 20% of the banks customers have low revenue potential and could be eliminated
- Our client needs to change the mix of products from group 1 and 2

- 30 -
Retail Bank
(McKinsey Quick on your Feet Competition)

Exhibit 1
Our client has 5 distinct groups of customers
Annual $ per customer1

Revenue Margin from Transaction % of total


Potential banking costs costs Cost to serve Profit clients

Working Draft - Last Modified 10/23/2011 8:45:44 PM Printed


Group 1 100 15 -5 -15 -5 30%

Group 2 100 50 -60 -15 -25 20%

Group 3 20 15 -5 -15 -5 20%

Group 4 100 50 -20 -15 15 10%

Group 5 100 75 -15 -15 45 20%

1 15,000,000 customers

SOURCE: Team Analysis McKinsey & Company | 0

- 31 -
Retail Bank
(McKinsey Quick on your Feet Competition)

Additional Questions to Steer Discussion


Questions for the candidate

5. What is the average annual profitability of a customer?

Average Customer

Category Profit Percent of clients Weighted profitability

Group 1 -5 0.3 $ (1.50)

Group 2 -25 0.2 $ (5.00)

Group 3 -5 0.2 $ (1.00)

Group 4 15 0.1 $ 1.50

Group 5 45 0.2 $ 9.00

Total N/A 1 $ 3.00

- 32 -
Retail Bank
(McKinsey Quick on your Feet Competition)

Additional Questions to Steer Discussion


Questions for the candidate

6. What is the annual bank profitability?

Total Profitability
Total Customers Average Profit per customer Total Profit
15,000,000 $ 3.00 $45,000,000.00

Note: If the interviewer decides to calculate each group out individually, push them to look
for shortcuts.
Our client has decided to institute a $.85 fee each month for all checking accounts. We have
advised them that they will lose a number of customers. We expect the following % of
customers to remain (read this chart to interviewee):
Percent of customers
that remain
Segment Percent
Group 1 60%
Group 2 60%
Group 3 20%
Group 4 60%
Group 5 70%

- 33 -
Retail Bank
(McKinsey Quick on your Feet Competition)

Additional Questions to Steer Discussion


Questions for the candidate

7. What is the new annual profitability per customer?


Note 1: If the interviewee is running out of time, help them along to ensure they get to
conclusion (e.g. ask them for their approach).
Note 2: Feel free to let the interviewee round off numbers here. Suggest $0.85 per month
should become $10 per year.

% of % of Profit
Customers customers from
who fit that will # of Profit per fees Total
each stay with remaining group (@$10 profit per
Segment Profit segment the firm customers (w/o fee) per yr) segment
Group 1 $ (5) 30% 60% 2,700K (13,500K) 27,000K 13,500K
Group 2 $ (25) 20% 40% 1,200K (30,000K) 12,000K 18,000K
Group 3 $ (5) 20% 20% 600K (3,000K) 6,000K 3,000K
Group 4 $ 15 10% 60% 900K 13,500,K 9,000K 22,500K
Group 5 $ 45 20% 50% 1,500K 67,500K 15,000K 82,500K

(Continued)

- 34 -
Retail Bank
(McKinsey Quick on your Feet Competition)

Additional Questions to Steer Discussion


Questions for the candidate

Total Profit 103,500K


# of Customers 6,900K

Average Profit per Customer $ 15

- 35 -
Retail Bank
(McKinsey Quick on your Feet Competition)

Suggested Solution and Structure


Solution Guide

You are walking down the hall and run into the CEO, he wants to know your recommendation:

A good answer includes, but is not limited to:

• Intro:
-The bank should institute a bank fee in order to meet the initial goal of increasing profits. This
is the quickest way to earn new streams of revenue, while segmenting out the unprofitable
customers. By instituting a fee you will be able to increase profit by 5x per customer on an
annual basis.
Note: The interviewee should include a detail or two on each group and how they are able to
increase profits (e.g. Group 3 was losing $3M per year we are now earning $3M in profit from them
on an annual basis).

• Risks
-Bad PR
-High transaction costs as people try to figure out if they are affected
-Estimates could be off
-Lose customers that could become profitable in the future

• Next steps
-Move forward with instating the fee
-Look at exempting certain groups from the fee

- 36 -
Baby Dinosaur (McKinsey)

Guidance for interviewer and


Problem statement narrative
information provided upon request

You are an MBA2 student, you walk into your  Dinosaur is the only one in the world, and it turns
apartment and you find a baby dinosaur in the corner out he is friendly. No other information is
of your room, what do you do? available.
 Candidate should layout some initial decision
tree that could include:
Take Action: run, call 911, call family/friends for
help, pick up a stick to hit it with, stay there and cry
Take No-action: Dinosaur leaves on its own,
Dinosaur is friendly, Dinosaur is not friendly and you
may die.

 If Dinosaur is friendly, student should think of


ways to monetize it.

Difficulty: Easy Industry: Other Type: Abstract, Decision Trees

- 57 -
Baby Dinosaur (McKinsey)

Structure to Steer Discussion


A strong candidate will be able to structure their thought process to include the following issues:
 Should they approach the dinosaur or run away? (depends on whether it is friendly).
 What can they do with the dinosaur? (Monetizing activities, vs. non-monetizing activities such as research,
etc.).
 When monetizing dinosaur, they should consider selling vs. building a dinosaur business/franchise.

Questions for the candidate

1. What are possible ways to monetize or make money in this situation? Sell it, or create business
around it. Student should brainstorm possible ways to create a business like using it for a movie, leasing it
to a zoo or an entertainment show, or creating an ecosystem around the dinosaur like a theme park.

1. What would you prefer to do, sell the dinosaur or use it for a business? Why? What are the
possible costs and revenues you can generate in each case?

2. What are the potential risks in this situation? Dinosaur dies, dinosaur attacks/eats spectators,
government seizes dinosaur and claims right to it, environmental concerns, etc.

3. What are the possible ways to hedge against the possibility of dinosaur death? Insurance, clone the
dinosaur, asking science experts in field for ways to take care of it, create an ecosystem around Dinosaur
like a theme park or having it star a movie.
- 58 -
Baby Dinosaur (McKinsey)

Conclusion

Recommendation Next Steps

A good recommendation will include creating  Next steps may include conducting a feasibility
an ecosystem around the Dinosaur such as a study around creating an ecosystem/theme
theme or entertainment park. It will touch on park.
other ways to hedge against the death of a
dinosaur, and potential risks like legal or
environmental risks.

- 59 -
Upscale Restaurant
McKinsey Final Round

Problem narrative Information provided upon request

Our Client is a upscale restaurant in TianJin, As China’s economy is booming, the upscale
serving government officials and high-level dining market is growing at 20% every year.
business customers. Its monthly revenue is 1.2 Customers for high-end dining are generally price
Million Yuan. The CEO recently hired McKinsey to insensitive.
help them increase profits.
All competitors are earning money. Competitors’
price and value proposition are similar.
Variable costs across industry is 50% of revenue.
Assume no fixed costs.
On weekdays, there is always a line for individual
rooms. As a result, the restaurant has to turn
away half of its customers due to capacity
constraint.

Difficulty: Hard Quant Heavy Industry: Hospitality Type: Profitability

- 89 -
Upscale Restaurant
McKinsey Final Round

Information provided upon request by Candidate

Individual Room : 20 tables Big Room : 20 tables

Week Day Weekend Week Day Weekend

Lunch Occupancy: 80% Occupancy: 30% Lunch Occupancy: 20% Occupancy: 30%
Price per person: Price per person: Price per person: Price per person:
150 100 100 100
Party size per table: Party size per table: Party size per table: Party size per table:
4 4 4 4

Dinner Occupancy: 100% Occupancy: 50% Dinner Occupancy: 30% Occupancy: 30%
Price per person: Price per person: Price per person: Price per person:
300 200 200 200
Party size per table: Party size per table: Party size per table: Party size per table:
6 6 4 4

- 90 -
Upscale Restaurant
McKinsey Final Round

Reason for low profits

Government officials and business customers prefer individual rooms to big rooms because of their
requirement for privacy. Currently our client is not meeting customer demand.

Question Solution

 Raising price.
What are potential solutions for this situation?
 Turning big room tables into individual rooms.

- 91 -
Upscale Restaurant
McKinsey Final Round

Question Solution

Through market research, we have determined that For weekday lunch, changing the price will result in
if we raise weekday individual room price by 33% , 10% customer loss.
we will lose 10% of customers. How will it change Previous Now
our profitability? Customer 4 x 20 x 80% = 64 64 x (1 - 10%) = 58
Price 150 150 x (1 + 33%) = 200
Revenue 64 x 150 = 9600 58 x 200 = 11600
Profit 9600 x 50% = 4800 11600 x 50% = 5800
Incremental Profit 5800 - 4800 = 1000

For weekday dinner, the underlying demand is


200% of current capacity, so raising price
WON’T reduce volume.
Previous Now
Customer 6 x 20 = 120 120
Price 300 300 x (1 + 33%) = 400
Revenue 120x 300 = 36K 120 x 400 = 48K
Profit 36Kx 50% = 18K 48K x 50% = 24K
Incremental Profit 24K – 18K = 6K
Daily Incremental Profit: 1K + 6K = 7K

- 92 -
Upscale Restaurant
McKinsey Final Round

Question Solution

A second solution is converting half of big room Cost


tables into 5 individual rooms. It will take 2 weeks Capital investment: 100K
for the restaurant to finish the decoration, during
which time the restaurant has to be completely shut Opportunity Cost: ~300K** (2 weeks of profits)
down. The decoration will cost 100K Yuan. What
is the total cost of this project?
*** Note: The observant candidate will quickly
calculate this from the initial revenue info given at
beginning of case rather than making heavy
calculations involved with calculating it from the
table of data.

Total cost = 400K Yuan

- 93 -
Acme Gas (1 of 9)
McKinsey, Round 1

Problem statement narrative Guidance for interviewer

Our client is ACME Gas, a national distributor of Liquid Use this question to assess the candidate's business
Natural Gas to people’s homes and businesses. The sense. Candidates can list a variety of aspects. Good
CEO is curious about the rise of alternative vehicle fuels candidates will explain why for each aspect listed. If
to gasoline and diesel. He is thinking of entering the they don't, push them to.
retail market for Liquid Natural Gas (LNG) filling stations.
He is asking for our help in deciding if ACME Gas should See sample response on next page
do this.

So, to begin with tell me what you expect the retail fuel
industry to look like?
Acme Gas (2 of 9)
McKinsey, Round 1

• Oligopoly of several major extraction firms that own retail outlets


• High number of retail outlets
• Small profit margins
• No product differentiation
• Price sensitive customers—may be only reason for choosing station
• Price is largely set by suppliers
Sample Response • No substitutes readily available to consumer
• Requires complete engine change
• Should see stable growth with economy
• Customers are made up of
• Trucking industry
• Personally owned vehicles
• Farm and commercial use implements, i.e. tractors or bulldozers
Acme Gas (3 of 8)
McKinsey, Round 1

Additional Question Guidance for interviewer

What areas would you explore to determine whether to Candidates should create a well-structured approach to
enter the market for liquid natural gas? look at both revenue and cost. Force candidates to dive
2 – 3 levels deep in the structure, to ensure that the
structure is relevant to the problem at hand and not a
common framework (e.g., 4 Ps, Porter’s 5 forces);
candidates should identify the buckets and provide a
brief rationale for each.

If candidate does not dive deep into costs, judges should


ask what cost areas would be explored as it is important
for next steps.
Acme Gas (4 of 9)
McKinsey, Round 1

• Size of Market
• Define the market • Financial implications
• Assess total market size • Potential revenue from addressable
• Isolate addressable market market
• Identify growth and relevant trends • Set up and ongoing costs
• Timeline to breaking even on investment
• Competition
• How many firms are in the potential • Company capabilities
market • What is our current technical ability to
• Likely reaction to entry meet this need (pumps, tanks, space,
• Likely reaction if no entry knowledge)
• Are we currently aligned with this market
Sample Response • Customer needs in terms of serving same customers
• Segment Customers • Can we make the required investment to
• Isolate drivers of purchase behavior enter?
• Define gaps in customer needs

• Barriers to entry
• Product differentiation
• Switching costs
• Brand loyalty
• Access to distribution and customers
• Legal aspects of moving to serve this
market
Acme Gas (5 of 9)
McKinsey, Round 1

Additional Question Guidance for interviewer

Question 1: How would you estimate the total markets Candidate should walk through the approach for
size for LNG retail vehicle fuel in 2012? calculating overall market size prior to sharing the
exhibit.

Total Market Size in Revenue = Vehicles * Tank size in


Question 2: Now let’s define the annual market size for Gallons * Fill Frequency * Price per Gallon
each customer segment in 2012 and evaluate if this is an
attractive market to continue looking. Next, ask the second question and provide the following
information (and Exhibit 1) upon request:
• Number of vehicles in US 2008: 100 Million
• Projected growth in vehicles: 20% over four years
• Current number of LNG cars: 1% of total
• Estimated price for LNG per Gallon in 2012: $2.00
Acme Gas (6 of 9)
McKinsey, Round 1

Customer Segment by Vehicle # of Vehicles in Tank Size Filling Frequency


Type (2012 projections) Segment (%) (Gallons) (Fills/Week)
Commercial Trucks 25% 100 3

Privately Owned Vehicles 67% 20 2

Farm and Industrial Use 8% 30 1

Number of vehicles in US 2008: 100 Million


Projected growth in vehicles: 20% over four years
Current number of LNG cars: 1% of total
Estimated price for LNG per Gallon in 2012: $2.00
Congress has mandated that 10% of vehicles be retrofitted to accommodate LNG by 2012
Acme Gas (7 of 9)
McKinsey, Round 1

Number of vehicles in 2012 = 120% of 100M = 120M


Number of vehicles that’ll be retrofitted for LNG as per Congress mandate = 10% of 120M = 12M

Total Market Size in Revenue = Vehicles * Tank size in Gallons * Fill Frequency * Price per Gallon
Market size Annual revenue per segment:
calculations0 Commercial Trucks: 25% of 12M * 100 * 3 * 2 = $93.6B
Privately Owned Vehicles: 67% of 12M * 20 * 2 * 2 = $33.28B
Farm and Industrial Use: 8% of 12M * 30 * 1 * 2 = $3.12B

Total annual revenue: $130B

• Initial Investment
• Land
A strong • Equipment investment
interviewer will • Construction costs
note that this is an • Permits, if any
attractive market, • Operating Costs
• Labor
but costs have to
• Facilities
be considered0 • Utilities
• Costs of goods sold (fuel and food/drink)
Acme Gas (8 of 9)
McKinsey, Round 1

Information provided upon request:


Expected market share: 20%
Assume initial of Net Profit Margin: 10%
$15B and required
Calculations:
payback of 5 Net profit per year = 10% of 20% of $130 Billion = $2.6 Billion
years. Can client Total profit over five years (ignore discounting) = $ 2.6 Billion X 5 = $13 Billion
achieve this
benchmark? Hence, the client cannot achieve this benchmark.

Increase revenues Decrease Costs:


• Increase price of fuel •Partner with existing stations to decrease
A strong • Sell more goods (convenience stores) investment—sell via license
candidate would • Sell more services (LNG specific vehicle •Franchise the format to distribute
infer the “so service) investment burden
what?” i.e., what • Sell other fuels •Lease land instead of purchase
• Carwashes and other services
should the client
• Sign volume deals with businesses and
do to achieve their municipalities to increase market share
payback goal? • Change location mix for stations to better
target
Acme Gas (9 of 9)
McKinsey, Round 1

Recommendation: ACME Gas should enter the market for retail LNG filling stations.
CEO of ACME Gas
•Growing market has increasing government support and is currently not served
is about to enter •ACME has the capability to serve this need
the room for a •ACME must make strong steps to minimize costs of both investment and operation in this
quick update on capital-intensive business. We can help with this key issue.
the project; what
would you tell Possible Risks: Drastic decreases in cost of substitutes, disruptive technology, and amendment to
government goals/legislation
him?
Next steps: Refine the marketing plan of the client to best achieve market dominance
Airport Parking (1 of 4)
McKinsey, Round 1

Problem statement narrative Guidance for interviewer

Our client is a provider of parking services in major metro The candidate should present a structure and walk
areas around the United States. The CEO recently was through some ideas. This is a brainstorm so numbers are
driving to the DTW airport and noticed a rise in private not necessary at this point.
providers of parking. She thought to herself, should we
enter this space? She has asked you to help her think
about this and wants to brainstorm with you.
Airport Parking (2 of 4)
McKinsey, Round 1

• Market Size (we will come back to this)


• Existing customer base
• Industry growth
• Competition
• Existing parking structures (local at airport)
• Other offsite parking
• Public transit (train, taxis, shuttles, etc)
• Existing Capability
Candidate should • What are the differences between running urban parking lots from those at an airport?
touch upon the • Busing/ shuttle system to drive customers to and from the airport
following • Is customer service a major factor?
elements in the • Revenue
structure0 • Number of spaces sold
• Add on services (car wash, newspaper sales, etc.)
• Pricing (consider probing about unique pricing – frequent park programs, packages, etc.)
• Costs
• Land (rent or own)
• Labor (bus drivers, ticket attendants, maintenance and security)
• Insurance
• Promotion (e.g. coupons)
• Fuel
Airport Parking (3 of 4)
McKinsey, Round 1

Provide the following facts as the right questions are asked but do not give them away freely:

• Flights/day 2600
• Average passengers/flight 80
• % of fliers that drive to the airport and hence need to park at the airport 20%
Candidate should • Average parking stay 3 days
have mentioned • Average price/day for parking $10
market size in the
framework.
Solution:
Ask the candidate • Total Passengers/Yr = 2600 X 80 X 360 = Approx. 75M
to estimate the • Divide by 2 to avoid double counting departing and arriving passengers = 37.5M
market size for
say O’Hare in • Airport parking market size
Chicago 0 • Passengers/Yr = 20% of 37.5M = 7.5M
• Parking Days/Yr = 7.5M X 3 = 22.5M
• $/Yr = 22.5M X $10 = 225M
Airport Parking (4 of 4)
McKinsey, Round 1

• Decline in airline travel growth caused by


• Higher fuel prices
Brainstorm • Security concerns (e.g. terrorism)
possible threats • Substitutes (high speed rail network)
to the
attractiveness of • Alternative travel to the airport such
this opportunity0 • New public transit
• Improved ride sharing possibly the result of social networking

• Number of competitors and their response to client’s entry into the market

There is about $225 million of parking business at O’Hare airport, one example of a major
metropolitan airport. If we are able to take advantage of our knowledge gained from urban
parking, this large, and probably growing, market could present a real opportunity for our
Possible summary growth. We need to look carefully at the competitive landscape and the economics at each
airport location and evaluate local competition and costs. From today’s analysis, this appears an
opportunity to explore further.
European Postal Service (1 of 8)
McKinsey Germany, Round 2

Guidance for interviewer and


Problem statement narrative
information provided upon request
Imagine you are consulting the CEO of a European ***This case was given by a partner with extensive
postal service company. He is wondering what future industry knowledge. It is a brainstorming case.***
trends to expect for his business and his industry and
how he should position his company. We are not talking about a specific company, but
What would you tell him? generally about a postal service provider operating in
Europe. Generally speaking a US provider faces similar
issues.

Very specifically to Europe is the fact that previously


existing national regulations have been lifted. Any
provider can now operate in almost every market or will
be able in the future. Direct competitors are usually
parcel services like UPS or service providers entering
from other countries.

The CEO is interested in the big picture of the market


and does not care for detail analysis of a specific issue.
European Postal Service (2 of 8)
McKinsey Germany, Round 2
Although this is a McKinsey case, a good candidate would drive through the first four steps on its
Case Outline own. The interviewer is looking for a high level approach. The last two steps need to be initiated by
the interviewer to guide the discussion appropriately.

Candidate Framework “Understand Business” “Identify Issues/Threats”


Framework could include: Tell the candidate to start Let the candidate identify all
•Understand current Business brainstorming about the business. potential issues with the business.
•Threats (Revenue/Cost issues)
•e.g. Internet, fuel, wages Help grouping and focus the case
•Opportunities on traditional business for a postal
•New products service provider.
•New markets in other
countries (organic/M&A)
•Or just classic industry framework

Opportunities Value chain Service Example


Let the candidate identify potential Let the candidate develop his idea Let the candidate show his
opportunities in the traditional of the value chain for Business business judgment (no detail
business model. Letters, Marketing and numbers required) whether it
Newspapers/Magazines. makes sense for the provider to
Although many ways are possible, deliver magazines/newspapers on
let the candidate reason for some Sundays
and then push towards value chain
integration.
European Postal Service (3 of 8)
McKinsey Germany, Round 2

In a first step the Questions the interviewer should ask:


candidate must try to What would you think the business looks like?
understand the Which products/services is such a company offering?
business of a typical What falls into this product category?
postal service What else would you expect?
provider

***Case Focus***Traditional Business: Letters and Packages


•First Class Letters
•Marketing mail
A good candidate will •Newspapers and magazines
address most of the •Packages
points. The
interviewer should Freight and Forwarding
help the candidate to •Ocean freight, air freight, industrial projects
group his/her ideas •Overland freight with full and partial loads using trains and trucks
into the defined
categories to Supply Chain Management
facilitate further work •Complete logistics for retail/industry clients including additional services

Financial Services
•Many postal service providers traditionally offer retail banking services and money transfers
European Postal Service (4 of 8)
McKinsey Germany, Round 2

Questions the interviewer should ask:


Next the candidate What problems do you see with the existing business?
should identify Do you think the traditional business has a future? How is it threatened?
risks/threats to the What do you think makes most of the volume: Private or Business customers?
existing traditional What would be a big business client?
business

Letters, related services , franking, philately


- Traditional letters are decreasing year after year
- Private business plays only a minor role
- Big customers like banks/telecoms/government start providing information electronically

Marketing mail
A good candidate will - Internet/E-mail marketing more and more replaces traditional advertising brochures
address most of the
points. Newspapers and magazines
-Editions for newspapers and magazines are decreasing, publishers go out of business

Packages
-No negative trends
European Postal Service (5 of 8)
McKinsey Germany, Round 2

Next the candidate


should identify
Questions the interviewer should ask:
opportunities for the
What opportunities do you see?
existing traditional
business

Traditional Business
•Business Letters, Marketing, Newspapers/Magazines
•Customers might outsource shipping (new customers – e.g. newspapers)
•Customers might outsource printing (new product)
•Packages:
•Increased internet shopping and increased business deliveries drive up volume
A good candidate will •Offer pick-up stations, self-service etc.
address most of the •Consider groceries shipping
points.
•General
•Cross border M&A or organic growth to use economies of scale
•*** This is a particularly interesting option since there is consolidation potential in the
industry, however this is not the focus of the case***
•Cost savings could include overhauled fleet, reorganization of distribution etc. ***Not
relevant for the rest of the case***
European Postal Service (6 of 8)
McKinsey Germany, Round 2

Questions the interviewer should ask:


Next the candidate What problems do you see with the existing business?
should identify Do you think the traditional business has a future? How is it threatened?
risks/threats to the What do you think makes most of the volume: Private or Business customers?
existing traditional How do you think the value chain for marketing material looks like?
business

Deciding to Designing Printing Shipping


Operating a
Promote a Promotion Promotion Promotion
business (e.g. retail)
product/service Materials Materials Materials
Interviewer A retailer like Identifying items to Layout etc. of the Printing of marketing Ship marketing materials
guidance for each BestBuy/Target who promote, selecting target materials materials
area (do not read needs to promote group
verbatim)

Remark In general the value chain for newspapers/magazines or business letters looks similar. There might be less integration opportunity.

Some of the European postal service providers have become the largest printers in their country.

Possible Currently the client is focusing only on shipping the materials. However it should be attractive to integrate other parts of the value
candidate chain. Printing is very promising, since this would yield large economies of scale. Possible is also to offer services to design
recommendation materials and provide data to the client to help him identify demographics/target groups.
European Postal Service (7 of 8)
McKinsey Germany, Round 2
Guidance for interviewer and
Additional Question
information provided upon request(1)
Given the high competition in the publishing market, a ***The candidate should show his business judgment***
publisher of magazines has approached the client to
distribute his magazines on Sunday instead of Monday. Things to consider (let the candidate brainstorm and
The publisher hopes to increase its subscriber base identify them)
since he thinks more people would have time to actually
read his product. Additional revenues through distribution of more
Our postal service client has previously not been magazines (how many percent? – e.g. 10% for this
operating on Sundays. Do you think it would be a good publisher, only a low number is reasonable).
idea to enhance his operation to Sundays? Potentially increase prices possible for other products
due to lower runtime.

Additional costs
Most of the costs for the service provider are fixed on a
given workday. It is not so much relevant how many
letters a postal service worker delivers in any given
street – its much more important that he needs to go
there at all.

Final Answer:
The client would increase its costs significantly (up to
16% since he would now operate 7 instead of 6 days).
His revenues would only increase minimally (e.g. 10% for
one of its customers).
European Postal Service (8 of 8)
McKinsey Germany, Round 2

Client CEO should consider the following options:


•Backward integration in the value chain
Final •Consider M&A of other postal service providers
Recommendation

•Other European service providers might act first in M&A and obtain an advantage
•Still existing regulation and “government organization culture” at potential targets might make
integration difficult
Risks

•Evaluate other European Markets to identify targets


•Create business case for a printing service offering
Next Steps
Online Bill Pay (1 of 4)
McKinsey, Round 1

Guidance for interviewer and


Problem statement narrative
information provided upon request
The client makes money every time someone pays a bill
through their online banking account. They earn
$0.50/Online Bill Pay and $1.00/Online Check (where a -The client is not a bank, but rather a third party who
physical check is sent to someone). manages this process for the bank and earns the state
amount/transaction
We know that 70% of People have online banking, but -The bank pays the transaction fee
only a small portion of this group actually pays a bill
through their online bank account.
-Core benefit to the bank is customer retention (e.g.
those that use online bill pay are more likely to stay with
They’ve come to McKinsey to help them figure out how the bank for many years)
they can increase the # of people using this feature.
Online Bill Pay (2 of 4)
McKinsey, Round 1

The structure developed can take multiple forms because the question is quite open ended.
One possible structure could include:
1. Market and Customers: understand the larger market and trends (what drives consumer
behavior, competitive landscape (both other banks and “substitutes” e.g. other online or non-
Structuring the online bill paying benefits)
Question 2. Customers with Online Bank Accounts (buckets within structure would address increasing usage
of online bill pay)
3. Customers without Online Bank Accounts (buckets within structure would address how to get
these customers signed up online)

Focus the candidate on CUSTOMERS. Ask what barriers there might be for customers
without online bank accounts?

Answers can include:


1. Perception that it is too difficult or complex to use / e.g. elderly population
2. Lack of internet access
Question #1 3. Loyalty to other online payment programs or auto debit
4. Lack of a bank account
5. Language challenges (population used to internet but customized websites, relevant for growing
Hispanic population)
6. Fear of security
(A strong candidate will list at least 4 of these).
Online Bill Pay (3 of 4)
McKinsey, Round 1
Why might someone with an Online Bank Account not use the bill pay function?
Answers can include:
1. Lack of awareness
2. Habit or loyalty to other online bill pay programs
3. Don’t pay any bills online – fear of security
4. Time required to input company names and information into banking account system (so that
Question #2 one can even pay the bills) is extensive
5. Bank with multiple institutions and like the flexibility of changing which bank they pay from (e.g. if
I write checks I can switch checking accounts every month if I want or if I pay my credit card at
amex.com I can store all my accounts there and select which one I want to pay from)
6. Language issues
(A strong candidate will list at least 4 of these).

5 YEARS AGO: TODAY:


-50% of the Population had Online Banking -70% of the Population have Online Banking
Additional Data on -10% of the Population paid Online Bills -20% of the Population pay Online Bills
Growth of Bill Pay Occasionally Occasionally
-5% of the Population were Active Online Bill -7% of the Population are Active Online Bill
Payers Payers

Ask the student what they notice about the data. Should recognize that:
1. Total people with online banking growing
2. Those using online bill pay occasionally has doubled
Question #3 3. Those becoming active bill payers growing at slower rate
Should extrapolate that while we are getting more people to pay occasionally, we are getting
less to convert into active users. This is bad news for the client.
Online Bill Pay (4 of 4)
McKinsey, Round 1
Ask Candidate why this could be happening?
Answers can include:
1. Decline in customer service or user experience
Question #4 2. Increased loyalty to other online payment systems
3. 5 years ago those engaging in any online bill pay were “early adopters” at cutting edge, and tech
savvy, more likely to convert at a higher rate, but this isn’t sustainable.
(A strong candidate will list at least 2 of these).

Let Candidate know it is clear that the Value Proposition needs to be improved. How?
Answers can include:
1. Sending out reminders for bills
2. Offering financing for bill payment
Question #5 3. Bundling all bills together with a “one click” payment system
4. Offering incentives to use online bill pay
5. Offering recommendations on better products once you see what the customer is paying each
month in bills
(A strong candidate will list at least 3 of these).

Answers can very but can include:


First focus on those with online banking, but not using online bill pay regularly.
1. Work on improving the value proposition and user experience so that those occasionally paying
bills become active bill payers
Final
2. Then work on converting all with online banking to online bill payers (must do step 1 above first
Recommendation because if this link is not improved first those converted from online bankers to occasional online
bill payers will never become active)
3. Figure out how to get more people into online banking – aim for 100% of those with bank
accounts to have online bank accounts
Opera House (1 of 4)
McKinsey, Round 1

Guidance for interviewer and


Problem statement narrative
information provided upon request

Our client, an Opera house located in a large -Opera house only does opera performances
metropolitan city, has been struggling with declining
profitability recently. They’ve hired McKinsey to help -Opera house does 6 unique performances (repeated for
them understand the source of declining profits and a few weeks each) throughout the year
provide recommendations on how to turn the situation
around. -Opera house has been around for 50+ years

-Customers are typically 45+ years old, highly educated ,


coming in from suburbs

Substitutes
-If they ask this question turn it around on them. Ask
what they think substitutes would be? Push them to list
at least 4 things. Then say “That sounds about right,
unfortunately I don’t have any information in this area”
Opera House (2 of 4)
McKinsey, Round 1
At this point, the Candidate should have developed a framework including the following buckets. The information can be
given in any order.

-Single show tickets: Night shows priced at $100, Sunday matinee priced at $70
-Club member ticket (all 6 shows of the year): Night package for $480, Matinee packaged for $300
-Group discounts (10 or more single shows): Night tickets for $80, Matinee tickets for $60
Products & Prices
*Night shows are Thursday, Friday & Saturday.
*Memberships are for single nights (i.e. Wednesday night membership)

Thursday nights: average of 300 customers


Friday nights: average of 450 customers
Saturday nights: average of 450 customers
Traffic/Volume Sunday matinee: average of 500 customers

*Capacity is 500 seats

When the student asks about costs, force them to state what they think the core costs of an opera
house should be. They should name things like:

Fixed (Management / Utilities / Rent or Mortgage / Marketing)


Costs Variable (Talent / Labor / Costumes / Equipment & Lights)

Tell them that the client has done some calculations that attribute fixed and variable costs on a per
show basis. They feel good about this figure as a proxy for the real costs of running each show –
this figure is $30,000.
Opera House (3 of 4)
McKinsey, Round 1
The student should do calculations to see if there are any unprofitable days. But they can‘t do that without knowing what
the mix of tickets is in each event. Tell them on average you see the following mix regardless of day of the week.

Single ticket: 70%, Group ticket: 10% , Club members: 20%

-Thursday = 100*(.7*300) + 80*(.3*300) = $28,200


-Friday = $42,300
-Saturday = $42,300
-Sunday = $33,500
Ticket Mix &
Profit/Day The student should quickly see that Thursday nights are unprofitable. Next they should move on
Calculations to question #2 – What do we do about it now?

Before they start brainstorming ideas, interject with a few facts:


- The client doesn’t want to let go of Thursday nights (and we shouldn’t because it still brings in
revenue, we should fix it)
- The client thinks the most opportunity is in the club tickets because it brings traffic to multiple
shows at once so we’ve decided to focus here, “How do we raise club ticket sales?”

Let the student brainstorm ways to raise club ticket sales, then stop them and say:

“An idea the client has is to allow customers to sign up for the club membership after viewing one
Ideas to Raise show via a single ticket. The idea is to attract more people after they’ve had a good experience.”
Ticket Sales (the client will give the customer their retroactive discount for the show they’ve already seen if they sign up,
but ignore this for purposes of the case). “The client has asked us to tell them what % of Thursday
single ticket purchases they would need to convert to increase Thursday night traffic enough to
break even?”
Opera House (4 of 4)
McKinsey, Round 1

The student should then solve for break even point first:
Money needed to break even = $30,000 - ~$28,000 = ~$2,000

Then they should find how many tickets it would take to break even:
Each club ticket is worth $80 in revenue, so $2,000/80 = ~ 25 tickets
Solution
The student should then find what % of the traffic that day is
25/300 = About 8 or 9% (ok answer)

Prompt the student to wrap up0

Answer:
- Opera house is losing money/declining profitability because they don’t generate enough traffic on
their Thursday nights. In fact they are losing money on Thursdays. Sunday is also not very
profitable, but does still generate profit for the client.

- The opera house can consider several strategies to turn this situation around. Possible ideas:
Discounting: discount tickets to maximize volume
Expand customer base: target local people who can make it on a Saturday night or professionals
Conclusion coming for a “treat after work”
Reduce variable costs: reduce staff or use only a portion of theater
Bundle tickets: Provide incentives for those buying Saturday tickets to one show to also pick up
Thursday night tickets to another show through bundling
Add or expand concessions: To further grow revenue

Risks/Next Steps: (answers can vary, but should be logical)


Pirates (1 of 10)
McKinsey Format, Round 1

Guidance for interviewer and


Problem statement narrative
information provided upon request

Our client is the captain of a well-established Pirate ship


in the Caribbean Sea who has recently noticed declining
booty reserves (aka cash). He’s realizing that his ship’s
current operations are not meeting his requirements for
a lavish lifestyle and is considering options for
generating additional booty. He’s asked you to
accomplish two important tasks:

•First, he wants to assess some of the reasons why his


booty reserves are declining; and
•Second, he needs to understand what he needs to do to
replenish his booty reserve.

So, the first question: why would booty reserves be


declining?
Pirates (2 of 10)
McKinsey Format, Round 1

The ship isn’t as profitable with its plundering; this could be due to declining revenues or
increasing costs:

• Revenue: Booty decreases could come from:


• Fewer amounts of ships to plunder
• Pirates have taken out all the potential ships
• Changes in the market have caused merchants to transition to other modes
of transport
• Increased safeguards against piracy
• Less revenue per plunder
• Ships are carrying less plunder,
Sample Response0 • Value of plunder (gold, etc.) is declining
• Other pirates have stolen from ships we’re attacking (latecomer)
• Less intimidating “yarr”

• Cost increases could be due to the following:


• Operating costs of ships could be increasing
• Several explanations – labor, food, maintenance, cannonballs, pirate hats,
parrots, etc.
• Less efficient workers on boat
• The captain is living a more lavish lifestyle
• Mutiny
• Debt payments are due on equipment
Pirates (3 of 10)
McKinsey Format, Round 1

Additional questions for candidate Solution guide

We’ve done an assessment of the ship’s booty and Revenues are increasing. However, it should also be
have found the following breakdown of revenues. noted that revenues are flat for the amount received
(Show Exhibit 1). What do you make of this? in the women category.

We have noticed that most of the areas of growth are


relatively steady, but that the market size women is
increasing with population.

Exhibit 2 should be given, showing the revenues from


each category.
Pirates (4 of 10, Exhibit 1)
McKinsey Format, Round 1
Pirates (5 of 10, Exhibit 2)
McKinsey Format, Round 1

REVENUES
Pirates (6 of 10)
McKinsey Format, Round 1

Additional questions for candidate Solution guide (See Next Page)

We have determined that, in order to increase


revenues at the scale necessary, growth in the
revenues gained from women need to increase at the
pace of all other revenues. With this information in
mind, calculate:

•The required incremental revenues in the sale


of women to meet this goal; and

•The new Growth rates for the ship in 1848 and


1849 given these new sales numbers for
women.
Pirates (7 of 10)
McKinsey Format, Round 1

Solution guide for Previous Page:

Growth of all other sectors from 1847-1848 is as follows:


(58,632,000 - 10,100,000) - (51,670,000 - 10,000,000) / (51,670,000 - 10,000,000) = 14.1%

Therefore, in order to meet this increase, the revenues from women must increase to (10,000,000 * 1.141) =
11,410,000 or incrementally by 11,410,000-10,100,000 = 1,310,000

New growth rate will be ((58,632,000 + 1,310,000) – 51,670,000) / 51,670,000 = 16.0%.

For 1848 to 1849, candidate should recognize that the growth should be included from the previous calculation

Growth of other sectors from 1848 to 1849 is as follows:


[(67,054,950 - 10,201,000) - (58,632,000 + 1,310,000 - 11,410,000)] / (58,632,000 + 1,310,000 - 11,410,000) = 17.1%

In order to meet this increase, revenues from women must increase from 11,410,000 (calculated in previous year)
to (11,410,000 * 1.171) = 13,361,110, or incrementally by 13,361,110 - 10,201,000 = 3,160,110

New growth rate will be [(67,054,950 + 3,160,110) – (58,632,000 +1 ,310,000)] / (58,632,000 + 1,310,000) = 17.1%
Pirates (8 of 10)
McKinsey Format, Round 1

Additional questions for candidate Solution guide

The captain has broken out his costs by successful Expected value of a “successful” plunder is:
and unsuccessful plunders. He has determined that,
on average, he spent 50,000 per successful plunder (500,000 – 50,000) * 40% - (70,000) * 60% = 138,000.
and 70,000 per unsuccessful plunder, respectively, in
1847. We also found that his average revenue per
successful plunder was 500,000. His plunders are
successful 40 percent of the time.

What is the expected value per plunder?


Pirates (9 of 10)
McKinsey Format, Round 1

Additional questions for candidate Solution guide

We have determined that, recently, he may be faced The candidate will now need to solve for the % of
with an increased cost of 100,000 per unsuccessful successful plunders and will need to use the
plunder. following equation:

Given this information, what percentage increase of 150,000 = (500,000 - 50,000) * (100% - x%) – (100,000) *
successful plunders must he undertake in order to (x%)
increase the expected value per plunder to 150,000?
Solving for x suggests that 46% of plunders will need
to be successful in order to increase profitability.
Pirates (10 of 10)
McKinsey Format, Round 1

Additional questions for candidate Solution guide

What would you suggest given the analysis and your The captain may need to increase his ship’s abilities
findings? which would involve some costs, but you may also
suggest we examine more lucrative opportunities
(such as shipping goods and services, targeted
plundering, expanding to new markets, etc.)
Power Plants (1 of 2)
McKinsey, Round 2

Guidance for interviewer and


Problem statement narrative
information provided upon request

Company generates several types of energy: coal, natural


The client is the CEO of a power company that does the gas, wind, nuclear and solar.
following things related to power: generation,
distribution, links regions with grids, buys energy This case focuses on coal power generation
wholesale and re-sells it, retails energy by selling it
directly to consumers. Lately, the client has experienced Revenue Data:
a profitability decline (about $100MM less than target) in • 10 power plants, each with capacity of 4.5MM mega
its core business of generating power and has hired watt hours per year and are running at 80% of capacity
McKinsey to diagnose the problem. • They sell each mega watt for $40

Costs Data:
• Coal related costs: $10/mega watt hour
• Fixed costs allocated across current capacity: $5/mega
watt hour
Power Plants (2 of 2)
McKinsey, Round 2
• Ask: If the client increased capacity utilization to 90% what impact would it have on profit?

Candidate should • At 80%,


identify capacity Revenue: 10 plants X 4.5 MM plant capacity X 80% utilization X $40/MWh = $1440M
utilization as a Variable cost: 10 plants X 4.5 MM plant capacity X 80% utilization X $10/MWh = $360M
possible issue for Fixed cost: Annual MWh produced X $5/MWh = $180M
profitability0 Profit = 1440 – 360 – 180 = $900M

• Similarly, at 90% capacity utilization, profits would increase by $135M

• Ask: Client would like to pursue the strategy of increasing utilization and feels that it will get
Candidate should them back to their previous profitable state. Brainstorm reasons why the plant has only operated
identify possible at 80% capacity to date?
reasons for low • A structured answer could include: Demand, Supply issues (equipment unavailable, technical
utilization0 staff unavailable, location/environment limits, lack of inputs), Company issues
(capacity/bottleneck in operations or untrained staff)

• Under -utilized capacity is one of the causes for reduced profitability.


• Recommendation: Increase capacity utilization.
Summary
• Next steps: Need to understand WHY plant operating under capacity. It could be several things
(list ideas brainstormed), but need to understand further.
Public School District (1 of 7)
McKinsey, Round 1

Guidance for interviewer and


Problem statement narrative
information provided upon request

The client has been struggling with declining funding ~100 schools
from the Ohio state government for the last 4 years.
Federal funds are practically non existent. Each year the A semi urban district close to a major Ohio district
school plans a budget, but when it receives its state
funding amount – the amount always falls short of the No private donors or funders
budget they’ve developed.

They want to find a way to systemically cut costs without


hindering the learning of their students.
Public School District (2 of 7)
McKinsey, Round 1
Ask the candidate where they’d like to start once they’ve developed their framework. A strong
Allow candidate to framework should touch on the big picture/current state (funding rec’d, target for cost cutting, etc),
develop their own but focus on the client’s cost structure grouping types of costs together, and also give high level
framework, then: consideration to the feasibility of certain cost reductions (e.g. union negotiations, publicity
impacts, etc)

Budget, funding and cost target: The client has been receiving $1.2 billion in annual funding from
the state government the past three years, while they have been building a budget for $1.35-$1.4
billion. Therefore they would like to cut between $150 and $200M from their annual budget.

Changes: No major changes to cost structure over the last four years

Fixed costs: The student should have named specific fixed costs. Provide information where
possible.
Upon request
Labor: Management staff (including benefits): $50M. Teachers: $80,000 each with a total of 4,000
reveal:
across the district. Principals and school leaders at $100,000 with a total of 2,000 across the district

Equipment: $500,000 per school (100 schools)

Building upkeep /support staff: $500,000 per school

Variable costs: Per student spending of $8,000 each; about 100,000 students
Public School District (3 of 7)
McKinsey, Round 1
After discussing “we have some interesting data on the students coming to our schools” and show them exhibit 1.
costs inform the
candidate that: Ask them what they see from this data.

Candidate should The population of school age children attending the public school system is going down, while
note that: charter school system is gaining population.

“I’ve got the data from this bar chart in a table too” and give Exhibit 2 to them.
Next, inform the
candidate that: Ask them what they see again.

The candidate should see that the total number of students in the area is declining. The population
is shrinking and people are moving out of the Ohio town.

If the candidate doesn’t get it on their own, ask them what they think when putting the two pieces
Candidate should of information together? They should see that the whole “market” for students is shrinking as is
their “share”. If the student doesn’t return to costs, ask them what they think the implication for
note that:
costs should be? The candidate should see that as students decrease over the last four years,
costs should also be decreasing, not just variable, but perhaps there are less teachers needed, etc.

The student should ask “Have there costs declined proportionally? The answer is no.
Public School District (4 of 7)
McKinsey, Round 1

Exhibit 1:
Students Enrolled
Public School District (5 of 7)
McKinsey, Round 1

Exhibit 2:

Public school system (the client) Charter school system Private school system
2005 105,000 5,000 10,000
2006 100,000 6,000 7,000
2007 95,000 8,000 6,000
2008 90,000 10,000 5,000
Public School District (6 of 7)
McKinsey, Round 1
Ask the
Where do the greatest cost cutting opportunities lie?
candidate:

They should see opportunity in teachers, school staff and then in the actual # of schools
(equipment and upkeep).

Teachers: The district strives for a ratio of 1 teacher to 30 students

Schools: The district strives for 1,500 students per school

Principal and school leaders: The district bases this off of the # of teachers and looks for a
proportion of 2 teachers to 1 school leader
Candidate should
Opportunity to cost cut
note that: 90,000 students/30 = 3,000 teachers (cut 1,000 teachers) = save $80M
90,000 students/1,500 = 60 schools (close 40 schools) = save $40M in equipment/building
upkeep/support
3,000 teachers/2 = 1,500 school leaders (cut 500) = save $50M

TOTAL SAVINGS = $170M (TARGET MET)

The student may consider some shut down or other transition costs or feasibility to the new plan
and want to take them into account. Listen and comment on them as good considerations and a
holistic view, but not necessary at this point
Public School District (7 of 7)
McKinsey, Round 1

Solution
Opportunity to systematically cut costs lies in aligning current downward student population trend
with cost structure.

- Specifically save $80M by adjusting teacher ratio


- Specifically save $40M through shutting down 40 schools
- Specifically save $50M through adjusting principal and school leader ratio

Candidate should Next Steps (can include)


wrap-up with: - Continue to watch and forecast student population declines
- Understand why public school losing share and learn from other systems to improve quality of
education
- Build transition plan for shut down of schools and lay offs (consider hiring PR experts to help
manage transition)

Risks (can include)


-PR backlash for closing schools and laying off teachers
-Union push back
Roadrunner Railways (1 of 7)
McKinsey, Round 1
Our client is Roadrunner Railways. They are a large railway company that ships freight throughout
Company the United States. Roadrunner is facing declining profitability and the CEO has approached us for
Background guidance.

Question #1: To begin, tell me Question #2: What areas would you explore to identify reasons for the
what you expect this industry declining profits? (Candidate should create well-structured approach
to look like? to look at Revenue & Cost. Force Candidate to dig 2-3 levels)
Small profit margins Revenue
• Little product differentiation • Changes in customer demand / operation bases
• Price sensitive customers • Customers switching to other freight transport methods (trucks, shipping, etc.)
(businesses) • Competition offering better prices or contracts
• New entrants who are leaner / more efficient and not burdened with legacy costs
Price competition: benefits to scale:
• Economies of scale throughout Cost
value chain (manufacturing, • Raw materials (Fuel)
marketing, sales, etc.) • Maintenance (Spare Parts, Frequency of Repairs)
• Labor (Train Crew, Maintenance Crew, Office Workers)
Oligopoly with a few large firms • Sales and Marketing (Freight Contracts requires Relationship Management)
• Management overhead
Should see stable growth with
economy

Substitutes such as trucks, boats,


planes
Roadrunner Railways (2 of 7)
McKinsey, Round 1
Based on this Exhibit, what areas would you explore?
Show Candidate
Exhibit A

Candidate should be able


Roadrunner Speedy Coyote Flash
to do basic mental math
normalize Roadrunner (% Total Gonzales (% Total (% Total
costs to industry average Cost) (% Total Cost) Cost) Cost)
percentages. Finance 7% 8% 10% 10%
Answer: Operations 48% 50% 52% 55%
Sales &
23% 10% 12% 15%
Candidate should then Marketing
identify Sales & HR 6% 7% 8% 6%
Marketing as an area for IT 13% 20% 14% 12%
further exploration. Other 4% 5% 4% 2%

Ask for Primary -Labor


Sales/Marketing -Advertising
-Travel Expenses
Cost Buckets

-Outsource vs. Insource


How Could -Increase Accounts/Employee (then Reduce # Employees)
Roadrunner -Relocate Sales/Marketing force to area where labor is less expensive
Reduce Labor -Use less costly workforce (e.g. College Grads)
Costs?
Roadrunner Railways (3 of 7)
McKinsey, Round 1
Currently the Sales & Marketing Department is outsourced to a firm that has centers across the country; and Roadrunner
pays for the allocated employees (called “Marketing Consultants”). Roadrunner did a comparative analysis within the
industry, and obtained the following labor stats.

# Employees:
-Outsource = 1,860
-In-House (Central Location) = 620
Cost Per Employee
-Outsource = $80K
-In-House = $120K

Why might In-House -Less employee turnover


Marketing/Sales -Increases specialization on Roadrunner-specific products/services
require fewer -Proximity to other functions
employees? -Increased opportunities for performance management

Candidate should either compare % Change or Total Cost. Either way, candidate should identify In-
What Can You House as the less expensive option.
Infer from this % Change : Total Cost :
Data? -Employees (-67%) -Outsource (1860 x $80 = $148,800K = $148.8M)
-Cost per Employee (50%) -In-House (620 x $120 = $74,400K = $74.4M)

Sales and Marketing department moved In-House due to cost savings.


What would you
recommend?
Roadrunner Railways (4 of 7)
McKinsey, Round 1
You’ve now dramatically rearranged the sales force, what are potential repercussions? i.e. What additional factors need to
be taken into consideration?
Note: Judge the candidate’s ability to brainstorm ideas. Following is a list of possible factors that should be considered.
Push them to mention the last point as it is important for the next steps.

-The cost of making such a transition (hiring new employees, obtaining office space, training, telecommunications, etc.)
-Revenue and client loss during the transition period
-Impact on relationships that the Marketing Consultants had built
-Customers may be dissatisfied that their sales person no longer visits them and only calls on the phone.
-If sales and marketing reps cannot visit the site, they may lose key insights into the customer’s business that cannot be
gleaned over the phone.
-Fewer employees in-house might not be able to have as many touch points with the various clients, so if that is important
to the customers it could lead to loss of customers and revenue (critical)

You mentioned revenue loss as a concern in the previous question, let’s explore that further.
Show Candidate Roadrunner did some predictive modeling of account retention and growth under two scenarios:
Exhibit B (1) Maintaining outsourced sales and marketing and (2) Moving sales and marketing in-house.
QUESTION: Calculate change in revenue if Sales/Marketing is moved In-House.

Change in
Revenue expected to Projected Revenue Projected Revenue Revenue Change in
Total Revenue be retained in-house (Outsource) (In-house) (immediate) projected revenue
1,200 900 1,320 972 (300) (348)
= 1,200 x 75% = 1,200 x 110% = 900 x 108%
1,000 950 1,080 1,140 (50) 60
= 1,000 x 95% = 1,000 x 108% = 950 x 120%
550 550 583 688 - 105
= 550 x 100% = 550 x 106% = 550 x 125%
Roadrunner Railways (5 of 7)
McKinsey, Round 1
What are your -Revenues for all segments will decrease in the immediate future from moving operations in-house
general observations -Growth for Medium and Small segments is higher with in-house operations, while that of the Large
of the data? segment is lower than with outsourced functions

What might be some A strong candidate will realize that the “Marketing Consultants” focused most of their efforts on
reasons that the the large segment, possibly due to the larger commissions these contracts entailed. Shifting
growth of the operations in-house will increase the contact with small and medium segments, while decreasing
various segments is that with the large segment.
affected differently
by having operations
outsourced?

What are the Answer: Based on these observations, there should be a reorganization of the workforce in the
implications? Sales & Marketing Department. Move the Sales & Marketing force that is responsible for Medium
and Small customers in-house, but continue to outsource the function for Large companies.

Roadrunner should reorganize the workforce in the Sales and Marketing department as a first step
towards increasing profitability. Specifically move the function in-house for the small and medium
segments, and leave the large segment outsourced as it is now.
-Currently, Roadrunner’s Sales & Marketing expenses are much higher than industry benchmarks.
Final -Moving operations in-house (Small & Medium) would reduce costs and increase revenue.
Recommendation -Functionality for the Large segment should remain outsourced.

Further analysis should be performed to more accurately gauge the cost of the transition and its
impact on operations, specifically focusing on potential integration issues, contract breakage fees
with outsourcing firm, ability to smoothly transition client relationships, and potential of the local
labor pool.
Roadrunner Railways (6 of 7)
McKinsey, Round 1

Exhibit A

Speedy
Roadrunner Gonzales Coyote Flash
(in $M) (% Total Cost) (% Total Cost) (% Total Cost)
Finance 140 8% 10% 10%
Operations 950 50% 52% 55%
Sales & Mktg 450 10% 12% 15%
HR 120 7% 8% 6%
IT 260 20% 14% 12%
Other 80 5% 4% 2%

2,000 100% 100% 100%


Roadrunner Railways (7 of 7)
McKinsey, Round 1

Exhibit B

Total Revenue
(in $M) # Accounts

Large 1,200 40

Medium 1,000 50

Small 550 75

Scenario #1 - Outsource Scenario #2 – In-House

% Expected % Expected % Expected % Expected


account retention Account Growth account retention Account Growth
(over 1yr) (over 1yr) (over 1 yr) (over 1 yr)

Large 100% 10% Large 75% 8%

Medium 100% 8% Medium 95% 20%

Small 100% 6% Small 100% 25%


TNT Funds (1 of 6)
McKinsey, Round 2

Guidance for interviewer and


Problem statement narrative
information provided upon request
Our client is TNT Funds, a small asset management firm.
TNT Funds manages investments in mutual funds from Candidate should layout a structure
wealthy individuals. Their account portfolio varies in the
average account balance and in the range of financial
assets invested. TNT Funds’ revenues derive from a fixed
annual management fee for their accounts.

TNT Funds is owned by a parent company, an insurance


company. The firm has 2 main sales channels – an online
platform, by which clients can buy and sell mutual funds,
and direct sales through agents of the parent company.

TNT Funds reported losses during the last 2 yrs and its
future projections are not bright either. They have hired
McKinsey to determine why they are not profitable and
what they can do to turn around their current position?
TNT Funds (2 of 6)
McKinsey, Round 2
Suggested Case Structure

incentives
Efficiency of sales
channels
Internal Factors methodology
pricing
Decreasing Revenues
customer trends
External Factors
competition
(rivals/substitutes)

IT investments

Overhead Increase in headcount

Office/Administration costs
Increasing Costs

commissions
Sales Channels
Transaction costs
TNT Funds (3 of 6)
McKinsey, Round 2

Ok. Now, TNT Funds would like to generate more revenue to balance their fixed overhead costs. By
how much should they aim to increase the average account balance of their clients in order to
achieve a 10% profit margin?
After the big
Consider the following data:
picture structure, Current average account balance is $5K
ask the candidate: Total fixed costs are $35M
Total funds managed are $5B
Variable cost is $20 per account managed
Management fee (revenue) is 1%

First, calculate the total cost per account:


Total # of accounts is 5B/5K = 1M
Allocated fixed cost is 35M/1M = $35 per account
Candidate should Total cost per account = variable + fixed = 20 + 35 = $55
determine that:
For a 10% margin, revenue per account will need to be 55/0.9 ~ $61
For a $61 revenue from a 1% management fee, average account balance should be 100 * 61 = $6100
That would require average account balance to grow by $1100 which is growth of 22%
TNT Funds (4 of 6)
McKinsey, Round 2

Next ask the In the mutual fund industry, most firms operate only one sales channel – either indirect (call center,
candidate: online) or direct (agents). Why would you think that is the case?

I believe companies tend to focus on one channel for 2 main reasons:

Customer focus: the 2 channels address different segments. High amount investors require
personalized service, whereas small investors require quick access to their invested funds and
should be ok with standard service.
Example answer:
Company: the 2 channels require a different set of capabilities from the company. The indirect
channel should be followed by an efficient operation or a good IT infrastructure (high fixed costs,
low marginal), whereas the direct channel would require deep understanding of mutual funds (low
fixed costs, high management costs).
TNT Funds (5 of 6)
McKinsey, Round 2
Now, let’s assume TNT was to focus on the direct sales channel to grow their customer base. What
% of the insurance clients they will need to capture in 5 yrs, if they wanted to manage 1% of the
mutual funds industry volume?
Next ask the
candidate: When asked, reveal:
-Total number of insurance customers is 4M and will remain as is
-Mutual funds industry volume is $1 Trillion and will grow at 10% CAGR
-Average balance of insurance customers account in 5 yrs will be $10K

First, calculate the volume of the mutual funds industry in 5 yrs (with 10% CAGR):

Current Year 1 Year 2 Year 3 Year 4 Year 5


1 TR 1.1 TR 1.21 TR 1.33 TR 1.46 TR 1.61 TR
Solution:
1% of this volume will be 1%* 1.61TR ~ $16 Billion (that’s the target volume)

# of customers needed to reach target volume is: 16 B/ 10K = 1.6 M

1.6 M is 40% of the total number of insurance customers (4M)


TNT Funds (6 of 6)
McKinsey, Round 2
Lastly: Based on our discussion, what would recommend the CEO of TNT Funds?

TNT Funds should recover from its poor profitability by focusing on the direct sales channel and
divesting the indirect efforts. The reasons are:

1- Through the direct channel they could leverage the parent company customer base and focus
on high $ investors

2- By focusing on one channel they could eliminate some of the fixed costs associated with the
Example Answer: indirect operations (a small company like them will have difficulties amortizing those costs by
achieving scale)

One the crucial next steps should be training the sales agents of the parent company to make sure
that they can serve the high-end customers well. Furthermore, TNT Funds needs to verify that the
sales agents are incentivized properly
Case 3: Retail Brokerage -hard- McKinsey&Company, Round 1

(Source: Case Interview from McKinsey&Company, Round 1)

Context:
Our client is a retail Brokerage firm with annual revenues of $5B. They are operating throughout the
US with 200 branches opened. Half of these branches are corporate and half are franchised.

Interviewer: What are the economics of this business?


A good answer will identify the followings:
In order to analyze the economics of the business I need to find more information about their revenues
and costs. Then I would like to look into what competition is doing on the market, how segmented the
market is and who are the consumers and what are their needs.
- On the revenue side, I need to understand where their revenues are coming from and then break it
into the two components price and quantity
- On the cost side, I would like to look at the fixed and variable costs:
o Fixed costs:
 SG&A
 IT system
 Marketing
o Variable costs:
 Labor
 Other cost related to commissions, fees
- Next thing that I would like to look into is what competition is doing: how many competitors do we
have, are there new competitors in the market, have they stolen share from us, are they offering
services that we are not
- We also need to understand how the overall market is doing: is it growing or shrinking; how
segmented the market is and if there are specificities related to regions
- Further, the consumer: who are they, what do they want how our client’s products meet their needs

Information to be provided upon request:

Revenues come from 2 different divisions:


- Trading : $3B
- Asset Management: $2B

Trading means that brokers do specific transactions as per their customer’s requests. The revenue in
this division would come from a fixed fee of $10 per transaction.
In the Asset Management division, the firm is administering the customer’s money and the revenues
come from a percentage from the total amount administered assets which is 1%

Costs:
Fixed costs: $1B
$800M – IT ( $700M from Trading and $100M from AM)
$100M – Marketing
$100M – SG&A

14
Case 3:Retail Brokerage -hard- McKinsey&Company, Round 1

Variable costs:
Trading:
 Commission to brokers: 40% of revenues
 Other costs: $2 per transaction
Asset Management:
 Commission to brokers: 40% of revenues
 Fee to an outsourcing firm that is managing the assets: 0.4%

Interviewee should now do the calculations for the profits:


Profits = Revenues – Costs

Trading division:
Revenues = $3B
Fixed Costs = $700M(IT) +$60M(Mkt.-pro rated from the revenues) + $60M(SG&A-pro rated from the
revenues) = $820M
Variable costs:
Commission: 40% of the $10 fee = $4 per transaction
Other = $2 per transaction
Number of transactions = $3B / $10 = 300M
Variable costs = $6 * 300,000 = $1.8B

Asset management division:


Revenues = $2B
Fixed Costs = $100M(IT) +$40M(Mkt.-pro rated from the revenues) + $40M(SG&A-pro rated from the
revenues) = $180M
Variable costs:
Commission: 40% of the $2B revenue = $800M
Fee for the outsourcing company = 0.4% of the $200B total administered assets = $800M
Total assets = $2B / 1% = $200B
Variable costs = $1.6B

Profits:
Trading profits = $3B - $1.8B - $0.82 = $380M (12.66%)
Asset Management profits = $2B - $180M - $1.6B = $220M (11%)
A good candidate will also observe that Asset Management is slightly more profitable than trading.

Interviewer: Now we are almost in an economic depression. What would happen to this firm if an
economic recession would happen next year? Calculate by how much they need to increase the
number of transactions/ assets managed now in order to breakeven in each division in case of
recession.

In order to do that I need to know what happened with this firm at the last recession in order to try to
benchmark the effects.
(if the interviewee does not ask for past recession effects ask to brainstorm on how they can estimate
the effects of the incoming recession till they get to this answer)

15
Case 3:Retail Brokerage -hard- McKinsey&Company, Round 1

Interviewer:
Number of transaction decreased by 50%
Assets managed decreased by 15%

For Trading:
If x = number of transaction needed now
#transaction * revenue per transaction = fixed costs + #transaction*variable cost per transaction
(50% * x) * $10 = $820M + (50% * x )*$6
x = 410M
They will need to increase the number of transaction by ~35% [(410M-300M)/300M]. I don’t think this is
feasible in a short period of time, especially just before a recession.

For Asset management:


If y= amounts of assets needed now
Amount of assets*% of assets = fixed costs for asset management + variable costs for asset management
(80% * y )* 1% = $180M + [(40% *1% *y*80%) + (0.4% *y*80%)]
y = $112.5B
There is no risk of becoming unprofitable in this division.

Interviewer: How can they address the risk of the recession (how can they keep the profitability at
current levels)?

They can either try to increase the revenues or decrease the costs:

Increase revenues:
- Change the product mix – get more asset management business because it is more profitable
o Advertise
o Incentivize brokers to get more assets
o Offer more benefits for customers coming to us instead of competition
o Extend office locations
o Offer new products for current customers
o Put in place a field sales team of brokers to get more assets or trade customers either by
attracting more and richer customers or by making the current ones put more money in
- Get more, richer customers
- Increase the fee per transaction or the percentage for the asset management
- Segment the market and differentiate depending on customer

Decrease costs:
- Fixed costs:
o IT seems to be the highest: outsource it because it can also bring some other benefits like
expertise from an IT firm, risk dispersion if it breaks down
o Use cheaper systems, less qualified labor
- Variable costs:
o Decrease the commission for brokers
o Get the asset management in house
o Link the commission of the brokers to the performance; create an incentive system to
actually make them bring more business
16
Case 9:Electronics Warehouse -medium- McKinsey&Company, Round 2

(Source: Case Interview from McKinsey&Company, Round 2)

Context:

Our client is an electronic warehouse selling all kinds of electronics and home appliances. It was
founded in 1990 and currently owns 375 stores located in all major cities across the US.
They have a healthy profit margin and represent a major player in the electronics retail market, but
the CEO hired us to help them grow even quicker.
Recently they opened a number of smaller conceptual stores and these stores are less profitable than
the regular ones.
We have the task to help them grow aggressively while maintaining the profitability.

Interviewer: What are the key areas to investigate in order to determine why the new stores are not
profitable?

A good answer will identify the followings:


I would like to look into the following areas:
- Revenues:
o Type of products sold in these stores
o Assortment
o Number of customers entering these stores
o Type of customers( income levels, family status, etc) and how the assortment in the
stores meets their needs
- Costs:
o As related to the volume sold ( mainly fixed costs)
o Labor costs; maybe higher trained personnel
o Distribution costs( from suppliers to the store)
- Competition:
o What is the presence of competition in the area
o What kind of stores the competition has in the area
- Other:
o Number of hours open
o Type of stores
o Location of the stores
o What are the customer’s needs and how our client manages to met them

Interviewer: How many stores do they need to open in order to secure a 20% market share in 5 years?

Information to be provided upon request:

Current electronic Retail Market = $150B


Current Market Share = 10%
Electronic Retail Market in 5 years = $200B
Aggressive growth would mean achieving 20% market share in the next 5 years.

34
Case 9: Electronics Warehouse -medium- McKinsey&Company,Round 1

(for simplicity of the calculation take into account that the current stores are all of the same revenue
size and the future ones will have the same average revenues; the candidate should realize that this
calculation would be different if that was not being considered)

Current state:
Market = $150B
Market share = 10%
⇒ Revenue = 10% * $150B = $15B
⇒ Revenue per store = $15B/ 375 = $40M

In 5 years:
Market = $200B
Market share = 20%
⇒ Revenue = 20% * $200B = $40B

⇒ Growth needed in revenues = $40B- $15B = $25B


⇒ No of stores needed = $25B / $40M = 625 ( but new stores will be only specialized that have
lower revenue)

Interviewer: Is their current strategy a successful one?


A good candidate will realize that the result is not feasible (they have 375 stores from 1990 – 18 years)

Interviewer: How can they achieve their objectives?

Possible options:
- Open only the old type of stores
- Choose locations with a specific type of inhabitants ( income, family status, hobbies, etc)
- Introduce new products and use the customer database to sell them
- Implement marketing campaigns, loyalty cards
- Make contracts with schools, institutions, hotels, etc
- Become a distributor for small electronics stores
- Raise prices on non price-sensitive products
- Acquire/merge with a competitor
- Get into other channels like online sales, door to door sales
- Start selling services (repairs, installations, etc)

35
Case 10:Grocery Chain -medium- McKinsey&Company, Round 1

(Source: Case Interview from McKinsey&Company, Round 1)

Context:

Our client is a grocery chain having 200 stores spread all over US. They have been enjoying good
profits and great results, though in the last 2 years they have seen a slowdown in the growth of the
market share and the profits and same store sales have gone down.
As a matter of fact, Wal-Mart has opened a store exactly 2 years ago and 3 more in the past two years
in the client geography.
Our client also started, 5 years ago, to open smaller stores closer to where there is a high density of
people.
We have the task to help them overcome their current issues.

Interviewer: What can be the reasons behind their low performance lately?

Elements to be discussed in the brainstorming session:

Market share decrease can be coming from several reasons:


- decrease in revenues due to customers spending less in our store and more in other stores,
- customers moving from our stores to another stores,
- an increase in the market size that is not captured by our client

Profit decrease can come either from revenue decrease or from cost increase
Decreased revenues due to:
- Competition: need to assess the entry of new competitors of significant investments from the
existing ones that might have stolen the consumers from our stores
- Change in consumer needs that were not spotted by ous client and have affected the traffic in
the store(e.g.: focus on organic products, need for additional services to be provided by the
store, etc)
- Change in pricing(specifically increase ) that might have decrease the spending of the consumers
or determined them to go to another competitor
- Change in the assortment of the store that might have determined some loyal customers to look
for those products in another place
- Change in the promotions that the client used to have(e.g. reduction of promotions, elimination
of loyalty programs, etc)
Increased costs due to:
- Opening of new smaller stores have a lot bigger costs per unit sold than the old stores
- A reduction in prices that provides lower profitability
- A change in the mix of products sold ( e.g. now the customer is selling more low margin
products)

Interviewer: How can they address the threat of Wal-Mart and the other competitors?
Information to be provided upon request:
• Wal-Mart carries 75% of the grocery items our client has at much lower prices
• Studies have showed that consumers perceive the prices in our client store as being higher
than both the ones in Wal-Mart and the ones in another close competitor

36
Case 10: Grocery Chain - medium- McKinsey&Company, Round 1

• The reality is that they are 20% higher than the ones in Wal-Mart and equal to the ones in the
other competitor
I do not think the Wal-Mart is a real threat based on the number of stores that they have yet (3 as
opposed to 200 of our client). But Wal-Mart can become a real threat once they expand.
Then it is very important to differentiate versus Wal-Mart. Our client will not be able to compete on low
prices with Wal-Mart but they can perform a market research and identify other needs that customers
have and Wal-Mart cannot provide and work on those attributes.

As for the other competitors, our client has to work on building its pricing image. There is clearly a
problem coming from the fact that they are perceived more expensive than the next competitor. Maybe
their prices are not following a good pricing strategy. They should probably conduct a price sensitiveness
analysis in order to properly identify the products that should have low prices and the ones that can
carry higher margins.

Interviewer: As a strategy to overcome their problems they decided to reduce the prices by 15%.
Based on the information below how many new stores should they open in the next 2 years to break
even?

Sales / store $40M


Stores older than 5 years 160
Stores opened in the last 5 years 40
Profit margin per store 30%

The sales/store for the stores opened in the last 5 years is lower than the sales from the older stores.
For the simplicity of the calculation we consider the stores equal.
(This information should help the candidate assess the validity of the price reduction)

Solution:

Sales / store after the price decrease


= 85% * $40M = $34M
Profit / store:
Before price decrease: $12M
After the price decrease: $10.2M
⇒ Profit loss = $1.8M

Total profit loss = $1.8M * 200 stores = $360M

No of stores to open = $360M / $10.2 =~ 35


(The number is not feasible as they only opened 40 stores in the last 5 years and these stores are
supposed to have fewer revenues than an average store)

37
Case 10: Grocery Chain - medium- McKinsey&Company, Round 1

Interviewer: They have performed another study to see the customer perception over a series of
factors. Based on this information, what do you think they should do?

Quickness

Customer service

Organic products
Factor tested

Fresh products
Competitor

Variety of products WalMart


Our client
Close to home

Promotions

Price

0 20 40 60 80 100 120

A good candidate will recognize that the closeness to home is the factor that they can work on as
our client is a better performer than its competitors.

It looks like on the fresh/organic assortment it has an advantage but a good candidate will recognize
that this advantage is easy to be copied by competitors

This should constitute a base for a recommendation for the client.

Example of recommendation:

Based on the findings so far, I think that for our client is critical to start concentrating on the attributes
where it has an advantage versus competition and that are also important to consumers (closeness to
home) and build its marketing campaign and future communication strategy on those identified
strengths. Another current advantage is the fresh/organic assortment but because it can be easily
copied, it will be difficult for the client to differentiate here. The risk is in the short term as building a
new brand image is not something that can be achieved very quickly but it will be critical to help them
recover the lost market share.

38
Case 12: Pharma Acquisition -medium- McKinsey&Company, Round 1

(Source: Case Interview from McKinsey&Company, Round 1)

Context:

Our client is a global pharmaceutical company that produces over the counter drugs and has its
headquarter in Frankfurt, Germany.
They are thinking of acquiring another pharmaceutical company located in San Francisco, that
produces nutritional drugs (for weight loss, diabetes, etc).
The CEO hired us to advise whether they should acquire the company or not.

Interviewer: What are the key areas to investigate in order to determine whether the acquisition is a
good idea or not?

A good answer will identify the following:


- First I would need to understand the rationale for the acquisition, that can be for:
o acquiring resources (increase capacity, increase distribution, broaden product line,
technology, human capital, R&D, brand name, customer base) or
o cost reduction (economies of scale, economies of scope).
It is very important that the acquisition makes sense economically (positive NPV), but we also need to
look into the organizational issues (will potential synergies be realized, is the firm in the position to
perform the integration).
In addition, I would assess the geographic differences of the two companies under discussion.
Finally I would look into the likely response of competitors if the acquisition occurs and maybe
alternatives to acquisition and compare it to the acquisition itself (other target, organic growth)

Information provided upon request:

• Purpose of acquisition: increase profits


• The SF company has 4 drugs in the market
• Both companies are selling their products globally.
• The R&D department is based in the same location with the HQ
• Revenues from an approved drug of the San Francisco based company is $1.5M

(show the next chart to the candidate)

42
Case 12: Pharma Acquisition - medium- McKinsey&Company, Round 1

Interviewer: We just discovered that we can improve the yield form phase 2 to phase 3 by investing
$150K in the R&D technology.
By how much should the yield increase so as to break even?

Other information given upon request:


The present value of launching a product is $1.5M

Solution guide:
To break even, cost needs to be equal to revenue
If x = the increase in the success rate from phase 2 to phase 3 then:
$150,000 = x * 70% * 90% * $1,500,000
=> x = 15.8%
(it means that the success rate should increase by approximately 15.8%/40% = 40%)

Interviewer: What are the risks involved with this acquisition?

(The candidate should be able to recognize the different risks involved)


• the strategic rationale
• likely response of competitors if acquisition occurs
• organizational issues: different locations for the HQ, integration of the two organizations
• profitability of the acquisition (NPV calculation)
• alternatives to the acquisition

43
Case 18: Mutual Fund -medium- McKinsey&Company, Round 2

(Source: Case Interview from McKinsey&Company, Round 2)

Context:

Our client is an asset management firm with flat revenue and profits. We have been asked to help
them to remedy this condition.

Interviewer: How would you think about this problem?

A good brainstorm should include discussions on the following elements:


- Revenue
- Costs
- Regulatory environment (e.g. restrictions on fees, etc)
- Competition (currently there is increased competition in funds)
- Distribution environment (e.g. the emergence on online brokerage)

Information provided upon request:

Revenue:
• $2 billion in revenue based on 1% management fee of $200B of assets

Costs:
• Overhead: $200M
• Money management: $800M, 50% based on assets
• Distribution/ brokers: $800M, 100% based on assets (i.e. 40% commission of total revenue).
All sales are made through independent brokers (they also sell funds from other companies).

Fund strategy
• US equities

Interviewer: At what level of assets will the fund owner begin to lose money?
(This is a question of fixed and variable costs)

Solution:

Currently our fund has a $200M in general overhead; I assume that is not going to change with asset
size.
The brokers are paid purely on commission, so all of their costs are variable.
The money managers are paid 50% on commission, meaning $400mm are fixed.

Total fixed costs = $200M + $400M = $600M


At a 1% management fee, we need a minimum of $60B in assets to cover our fixed costs

57
Case 18: Mutual Fund -medium- McKinsey&Company, Round 2

Interviewer: Based on what you have found, what would you recommend to the fund manager?

- We need to look at ways to drive revenue first. We could change the incentive structure for the
money managers, adjusted more to the fund’s performance.
- With the brokers, we could pay them a higher commission to try to encourage them to sell our
fund over another (we would have to tease out the quantity/price relationship to see if this
makes sense.
- There probably is not much to gain with overhead.
- Additionally, this is a US fund only. We should consider launching an additional fund- perhaps
focusing on international markets- to drive up revenues and leverage our brand.
- We are a $200B fund- we probably have a strong brand that we could use to sell more to our
existing customers (i.e. new fund) and possibly bring in new customers as well.
- Look into new distribution methods
- Look into asking brokers to do cold calls
- Have the brokers in house (this will increase fixed costs but might provide better productivity)

58
Practice Cases – Case 1: Great Burger McKinsey & Co.
Interviewer: The team started thinking about potential synergies that could be
achieved by acquiring HD

Here are some key facts on GB and HD.

Interviewer: What potential synergies can you think of between GB and HD? For
your information, a synergy is an area where additional benefits can be captured
over and above the sum of the two companies (such as cost savings or additional
revenue).

A good answer would include the following:


There appear to be opportunities in cost savings and in revenue gains.
In cost savings:
– There may be an opportunity to save on General & Administrative Expenses through
combining management locations/functions
– There may be decreased Cost of Sales (per unit) because the companies are purchasing
greater volumes together

28
Practice Cases – Case 1: Great Burger McKinsey & Co.
In revenues:
– Additional sales can be achieved through selling Donuts in GB stores
– Also GB has a greater global presence which HD could leverage in order to grow outside the
US

A very good answer might also include the following:


– GB appear to manage their property and equipment costs better, which means that they
may be able to transfer this skill to HD
– Since GB has greater Sales per Store, they may have better skills in finding good locations
for stores, and could transfer this skill to HD
– Since GB is bigger, it probably has more investment capital available to help HD grow at a
more rapid rate.

Interviewer: The team thinks that, with synergies, it should be possible to double
HD’s US market share in the next 5 years, and that GB’s access to capital will allow it
to expand number HD of stores by 2.5 times. What sales per store will HD require in
5 years in order for GB to achieve these goals?

You should assume:


• Doughnut consumption per head in the US is $10/year today, and is
projected to grow to $20/year in 5 years
• For ease of calculation, assume US population is 300m
• Use any data from the earlier table that you need

A good answer is as follows:


HD will require a sales per store of $1.2m
– Today’s market share is $700m/$3b = ~25%. This is available from the earlier table, and
you are encouraged to make sensible, round estimates in a calculation.
– Expected US market in 5 years = $20 *300m = $6b
– If HD double today’s market share, the will have a market share of 50%, so their sales will
be 50% x $6b = $3b
– They are also expect to have 2,500 stores (= 2.5 x 1,000)
– So sales per store = $3b / 2,500 = $1.2m

A very good observation to make is that this seems like a realistic growth target, because we
are requiring stores sales to less than double, while we already know that per head
consumption of donuts is likely to double.

Interviewer: One of the synergies that the team thinks might have a big potential is
the idea of increasing the businesses’ overall profitability byselling doughnuts in GB
stores. How would you assess the impact of this move on overall profitability?

A good answer is as follows:


I would try to work out the incremental impact this move would have on profits. To do this I
would:
– Calculate the incremental revenues we would get from selling donuts in GB stores (how
many, at what price, etc)
– Calculate the additional incremental costs that would be incurred from doing so (for example,
additional staff, additional training, additional marketing, additional distribution and
purchasing costs)
– I would also look at the additional store investment we would have to make (for example,
extra space, new equipment, etc)

A good answer would also include:


– We should also investigate if the additional donut sales would mean lower sales of traditional
GB products. For example, breakfast products might be affected as many people have donuts
for breakfast. In case you are unfamiliar with the term, this concept is known as
“cannibalization”.

29
Practice Cases – Case 1: Great Burger McKinsey & Co.
Interviewer: What would be the incremental profit per store if we think we are going
to sell 50,000 doughnuts per store at a price of $2 per doughnut at a 60% margin
with a cannibalization rate of 10% of GB’s sales? Note that the cannibalization to
sell 50,000 doughnuts per store at a price of $2 per doughnut at a 60% margin with
a cannibalization rate of 10% of GB’s sales? Note that the cannibalization rate is the
percentage of GB products which we think will not be sold because they have been
replaced by donut sales. Here is some additional information which will help you:

Current units of GB sold per store 300,000


Sales price per unit $3 per unit
Margin 50%

A good answer is as follows:


There will be $15,000 incremental profit per store:
– Donut sales will bring in an additional $60,000 in profit ($2 price x 50,000 x 60% margin)
– However, we will lose $45,000 in the original profit from GB sales (10% cannibalization rate
x 300,000 products x $3 price x 50% margin)

Interviewer: You run into the CEO of GB in the hall. He asks you to summarize
McKinsey’s perspective so far on whether GB should acquire HD. Pretend I am the
CEO - What would you say?

A good answer would include the following:


Early findings lead us to believe acquiring HD would create significant value for GB, and that
GB should acquire HD
– US Growth targets seem achievable given the expected growth in Donut consumption in the
US
– There are other opportunities to capture growth from international expansion of HD
– We also believe there are other potential revenue and cost synergies that the team still
needs to quantify

A very good answer might also include the following:


– We believe can HD add $15k in additional profit per GB store simply by selling donuts in GB
stores. This represents a ~25% increase in store profit from this move alone.
– We will also provide you with recommendations on the price you should pay for HD, as well
as any things you need to think about when considering integrating the two companies.

30
Practice Cases – Case 2: Magna Health McKinsey & Co.
• Mix of patients:
o Compare demographic data for Magna and Sunshine: should be easy to obtain
from Magna; a scan of the employee schemes covered by Sunshine should
give a good general picture of their demographic profile
o See if Magna's referral cost has increased in line with the change in
demographics of the subscribers

(Helpful Tip: In giving the answer, it's useful if you are clear about how the analysis you are
proposing would help to answer the question posed.)

Interviewer: Magna's CEO has a hypothesis that Magna is paying too much in
cardiology referral costs for its patient population. He asks the McKinsey team to
look at Magna's cardiac patient population more closely and tell him how many
referrals he should expect on an annual basis. Assume the following:

• Magna has 300,000 patients in any one year


• 20 percent of its patients are age 65 or older
• In the U.S., patients with serious heart disease visit specialists (cardiologists)
on average of five times per year

You should always feel free to ask your interviewer additional questions to help you
with your response. In this case, you should recognize the need to know the
prevalence rate of serious heart disease to complete this calculation. Once asked,
your interviewer would provide you with the following information:

• The prevalence rate of serious heart disease in the 65+ population is 30


percent
• The prevalence rate of serious heart disease in the under age 65 population is
10 percent

Based on the correct calculations, your response should be as follows: Magna should expect
210,000 cardiac referrals annually based on its patient population. You should have
approached the calculations as follows to arrive at that answer:

• 300,000 total patients


• 20 percent x 300,000 = 60,000 patients age 65+
• 18,000 x 5 = 90,000 referrals per year
• 240,000 Magna patients under the age of 65
• 240,000 patients x 10 percent = 24,000 patients under age 65 with serious heart
disease and 24,000 x 5 visits per
• year = 120,000 visits per year total
• 90,000 + 120,000 visits per year = 210,000 total Magna patient external cardiology
visits

Interviewer: When the team tells Magna's CEO that based on Magna's patient
population he should expect about 210,000 cardiology referrals a year he exclaims,
"We currently pay for 300,000 annual cardiology referrals for our patient
population!"

Why might Magna's annual cardiology referrals be significantly higher than U.S.
averages? What would you do to try to verify if any of these were a key cause of this
problem?

33
Practice Cases – Case 2: Magna Health McKinsey & Co.
There are a number of answers to these questions, and you are on the right track if your
responses included some of the ones below:

• The prevalence rate of heart disease in Magna's patient population is higher than
average. To see if this was a cause of the problem, McKinsey should audit the internal
data on heart disease prevalence and compare it to US National data.
• Magna's primary care physicians are referring patients who do not have serious heart
disease to specialists. The team should interview specialists to get their opinion, or
follow through a sample of patients who were referred.
• Primary care physicians are not comfortable (e.g., they are poorly trained or
inexperienced) treating cardiac patients, even those with minor problems; they want
to avoid malpractice suits. McKinsey should interview Magna physicians and institute
an external review.
• Magna doesn't have clear guidelines on when physicians should be referring patients
to specialists (or if guidelines exist, physicians are not complying with them). The
team should gain an expert opinion on the current guidelines to see if this was a key
cause of the problem.
• There are no incentives or penalties to prevent physicians from referring patients with
less serious problems to specialists. In order to verify this is a key cause of the
problem, the team should review incentive schemes if they exist. They should also
compare similar companies/situations (e.g., prescription control mechanisms, etc.).

Interviewer: At this point in the study, you bump into Magna's Head of Health
Services in the corridor. He is responsible for all matters related to the provision of
services to subscribers, both inside and outside the Magna Network. He asks you if
you have made any progress. How would you respond?

The ability to come to a logical, defensible synthesis based on the information available at any
point in an engagement is critical to the work we do. Even though we'd consider ourselves to
be early in the overall project at this point in the case, we do want to be able to share our
current perspective. One ideal answer would include the following points:

Findings

• We have investigated all the drivers of profit for Magna. Although there is likely to be
room for improvement in a lot of areas, it seems the claims cost is a big area for
improvement.
• Relative to the market and to competitors, Magna seems to have high claims cost per
patient. Our initial indication is that there may be highest room for improvements in
the cost of referrals outside the network.
• There are a number of reasons as to why this may be happening (list as in previous
question).

Next Steps

• We are working to pin down the most significant reasons why Magna has high claims
cost per patient.
• We are going to be looking into other areas such as reduction potential in other costs,
as well as improvement potential in terms of premiums or other sources of revenue.

Interviewer: After some additional investigation, your team decides that changing
the behavior of Magna's primary care physicians has potential to reduce cardiac
referral costs while maintaining high quality care. The team believes that
introducing some sort of incentive plan for physicians might help reduce the referral
rate. You propose the following pilot plan:

34
Practice Cases – Case 2: Magna Health McKinsey & Co.
• Magna pays bonuses of $100,000 per year to each of the 10 primary care
physicians with the lowest cardiac referral rates consistent with good patient
outcomes.
• Magna increases overall fees paid to primary care physicians to handle more
of their patients’ basic cardiology needs. Overall fee increases would total $1
million.

How many fewer cardiology referrals will Magna need to have in order to recoup the
cost of the pilot incentive plan? For simplicity’s sake assume:

• The cost of a cardiology referral is $200.


• Magna currently has 300,000 cardiology referrals per year.

If the incentive plan reduces cardiology referrals by 3.3 percent or 10,000 referrals, Magna will
recoup the cost of the incentive plan. One potential approach to the calculation:

• $1 million + (10 * $100,000) = $2 million for incentive plan


• $2 million/$200 =10,000 referrals
• 10,000 referrals/300,000 total referrals = 3.3 percent reduction would pay for
incentive program

Interviewer: Your team projects that the incentive plan has the potential to reduce
referrals by 5 percent in its first year, and an additional 2 percent in its second year.
If these projections are correct, by how much would Magna's referral costs be
reduced over a two-year period with this program?

Referral costs would be $4.14 million lower in the second year. Over the two years Magna
would save $7.14 million. One potential approach to the calculation:

Year 1 Savings with Program

• 300,000 total referrals


• 5 percent reduction in referrals = 15,000 referrals
• 15,000 x $200 = $3.0 million in savings in year 1

Year 2 Savings with Program

• 285,000 total referrals


• 2 percent reduction in referrals = 5,700 referrals
• 5,700 x $200 = $1.14 million in savings
• $3 + $1.14 = $4.14 million in savings

Therefore, total cumulative savings over the 2 years = Year 1 savings + Year 2 savings =
$3.0m + $4.14m = $7.14m.

Interviewer: Your team presents its physician incentive proposal to Magna’s CEO.
The CEO, in consultation with his Medical Director, agrees that this is feasible and
says that they will pilot it for cardiac referrals.

At the end of the meeting the CEO says, "I like the work you’ve done, but it's not
enough to address our current financial situation. Physicians are professionals who
care deeply about patient care and I think there's a limit to how much cost we can
expect to reduce utilizing financial incentives exclusively. Besides cardiac financial
incentive programs, what other ideas should we consider to reduce the cost of
Magna's specialist referrals?"

35
Practice Cases – Case 3: Granite Investments McKinsey & Co.
A good answer will identify the following options:

- Increase the number of year you use a PC (to 5 years)


- Use less PCs (<2000)
- Buy cheaper computers (or have a lease with computer manufacturers)
- Have a lower service level requirement (so you do not need 10 support people)
- Ask Apogee to hire cheaper technicians for them
- Do competitive benchmarking to see if you can use another vendor (need to ensure
that you can break the existing contract) – even outside the US

Interviewer: Great. OK, one of the systems that they have outsourced is the website.
Now, Apolgee has proposed that the client increase its service level from 99% to
99.9% for $5M/year. Should they do it?

Interviewee: It will depend on whether doing so is profitable to our client. Our client can
estimate the incremental revenue due to increased service level and compare that to the
incremental cost ($5M).

Data only provided when asked


• Number of total transactions on the website/year = 200M
• 80% - general inquiries (although these might impact brand etc., and lead to
more revenues – but we will ignore that for now)
• 20% - revenue generating
• Average revenue/transaction = $30

Interviewee should calculate the following:

Total revenue from the website (if 100% service level i.e. no transactions are lost)
= 0.2 * 200M * $30 = $1200M

If they gain 0.9% of the transactions that they now lose = 0.009 * $1200M= $10.8M

By increasing service level by 0.9%, the client gains $10.8M. If they have to pay Apolgee $5M,
they would still make a profit of $5.8M (assuming no other costs). Hence, they should increase
the service level as suggested by Apolgee.

Interviewer (Role-Play): Imagine I am the CEO of Granite Investments. I am very


frustrated with the Apolgee – that their prices are so high, and now they are asking
me to pay another $5M. As a McKinsey associate on this project, I look to you for
help. What would you tell me?

Interviewee: Mr. CEO, we analyzed Apolgee’s proposal to increase the service level to 99.9%.
Our analysis shows that by getting the extra 0.9% revenue generating transactions you would
gain $5.8M in profit. Therefore, we recommend that you take the deal. In addition, because
Apolgee’s profit margins seem very high at 34%, in the next a few days we would like to do
some competitive benchmark analysis to see if other vendors offer better prices with the same
level of services to see if we are overpaying Apolgee. If yes, we can renegotiate contract
terms with Apolgee or switch to other vendors. Of course, we need to review the contract to
see if there are any restrictions we should be aware of. If we want to switch, we need to
assess the impact of switching cost.

Interviewer: In fact, Apolgee has agreed that we can break the contract anytime we
find better prices outside.

Interviewee: Great! Then we shall do the benchmark exercise and see if we can find better
vendors.

38
Practice Cases – Case 4: Meat Processing McKinsey & Co.
(Source: Interview case from McKinsey Round I, 2006)

Context
Our client is a meat processing company. They buy livestock, process and then sell
to major retail supermarket chains. In early 90’s, the industry went through
consolidation and now there are only 5 big players in the market. During this time
our client bought about 8 regional/local players in the market who were not doing
well and had old assets that had depreciated.

The CEO has called McKinsey because their profitability is declining and they want to
understand what they can do.

Interviewer: What are the key areas you would look into?

A good answer will cover the profitability structure

I would like to know if the revenues are declining or the costs increasing or both.

On the revenue side, I would look at the changes in the two drivers of revenue – price and
quantity. Before that, I want to understand how the overall market is doing – is it growing or
shrinking (however, I think this would not be a problem). The next thing I would look into is
what the competition is doing – are there new competitors in the market – have they reduced
the prices or stolen share from us. I would also look at customer trends – for instance, end
consumers could be moving towards healthy foods or vegetarianism. Further, the customers
i.e. the supermarket chain, might not provide enough shelf space since their bargaining power
might be higher or might squeeze our client on the price. These factors would affect the price
and quantity we sell.

On the cost side, I would look at the fixed costs and variable costs of the facility.

Fixed costs:
- Plant/Facility – Rent
- Equipment for processing
- Insurance
- SG&A

Variable costs:
- COGS
o Direct materials such as birds (quality of birds/type of birds) and chemicals
needed for processing
o Direct labor
o Production costs
- Transportation cost (was told that each facility is independent and complete so no
transportation is required within facilities)

Interviewer: The processing costs are high for the client. We did competitive
benchmarking of our “birds” division with King Chicken. Data provided.

Client King Chicken


Average quantity of birds 300,000 500,000
processed each year in each plant
Average processing cost/bird $30 $20

Interviewer: What do you think?

A good answer may identify the following:

The processing costs are a lot higher compared to King’s chicken. Reasons could be:

39
Practice Cases – Case 4: Meat Processing McKinsey & Co.
- King Chicken sells more chicken and has reached economies of scale, and therefore
has lower fixed cost per bird.
- King Chicken has better capacity utilization which leads to lower overhead per bird.
- King Chicken has more efficient or cheaper processing due to newer equipments or
more advanced technology.
- We process different types of birds that might require extra care or more specialized
process or more specialized labor leading to higher costs
- We have a better skilled labor force as we deliver premium products.
- Since our average processing cost is higher, it could be that some of our plants have a
lot higher processing costs which bring up the average cost.

Interviewer: One of our Associates came running in and told us that one of their
plants (out of 10) processes turkey instead of chickens, whereas King Chicken only
processes chicken. The processing costs in that plant are a lot higher. He found out
that it costs twice the amount to process a turkey than it takes to process a chicken.
Can you find out what the costs for processing a chicken and a turkey are in our
plants?

Turkey Chicken
Quantity of birds processed each 600,000 266,000
year in each plant
No. of plants 1 plant 9 plants

Interviewee should do the calculation:


I would just take the actual costs of processing of both turkey and chicken, and equate it to
the total average costs of processing (which we know from the data earlier) in all 10 plants.

Lets say the processing cost of chicken = $X


Then, processing cost of turkey = $2X

600,000 * $2X * 1 plant + 266,000 * $X *9 plants = 300,000 * $30 * 10 plants


=> X = $25

The cost of processing chicken is $25 and that of turkey is $50. The cost of processing chicken
is still higher than our competitor’s cost of $20.

Interviewer: Why do you think? What would you think of doing?

A good answer should include the following:

The cost of processing is still higher by $5 than our competitors. That might not be good. If we
produce the same type of meat as our competitors, given the commodity nature of this type of
meat we have to take lower profit margin to stay competitive. If our competitors decide to
reduce price to gain more market share, we will be very vulnerable.
If we produce premium product, the supermarket chains might not be willing to pay much
higher price to buy our product as I would think that $5 might be a significant % of the total
cost of meat’s tag price.

If our client’s product is a branded premium product and consumers are willing to pay the
extra dollars for better taste, packaging or healthy meat (whatever is the criterion that
differentiates it), higher cost structure may be fine. However, I am not sure how much extra
price consumers will be willing to pay for premium chicken or turkey meat. Let’s say that one
bird produces two pounds of meat, which means consumers will have to pay $2.5 extra per
pound. It seems very high. Therefore, we should still find ways lower costs. If our product has
the same quality as our competitors’, it’s even more important to look at ways to reduce costs.

To reduce both fixed costs and variable costs,


- We could look into consolidating the plants, especially if they are underutilized. That
might bring down costs as we can consolidate both facilities and equipments.

40
Practice Cases – Case 4: Meat Processing McKinsey & Co.
- If we haven’t don’t so, we could look for opportunities to maximize synergies as we
acquired so many players, such as consolidating purchasing to have better price from
suppliers
- We could share best practices across different facilities.
- Improve labor productivity or hire cheaper labor without affecting quality of work
- Find better suppliers to lower material costs.
- Buy cheaper equipments or lease them instead.
- Expand to other markets such that capacity utilization increases, or lease/sell the
unused capacity.
- Stop processing turkey to reduce average processing cost.

41
Practice Cases – Case 6: Sugar Cereal BCG
(Source: Sample case from BCG website)

Context
Your client is the sugar cereal division of Foods Inc., a U.S.-based distributor and
manufacturer of packaged foods. According to the division president, Foods Inc.'s
traditional strength has been with grocery stores, which still account for the
majority of its $1.1 billion in sugar cereal sales. But Big M Mart, a discount chain, has
been growing at a healthy rate of almost 15 percent per year and has now become
Food Inc.'s largest customer. Your client is not sure how to react, and has asked BCG
for assistance with its distribution strategy

Establish understanding of the case

Interviewee: First, let me make sure I understand the problem. Our client specializes in
sugar cereals traditionally distributed through grocery stores. Sales to Big M Mart, a discount
chain, have been growing at 15 percent per year, and the chain has recently become the
largest distributor of the client's product nationwide. We are here to help evaluate the
distribution strategy in light of Big M Mart's growth.

Interviewer: That is correct.

Interviewee: Could you explain to me how grocery stores differ from discount stores?

Interviewer: Sure. Grocery stores generally specialize in food, as well as selling


some household goods and over-the-counter pharmaceuticals. Discount stores, on
the other hand, offer food alongside a wide variety of merchandise, including
clothing, home electronics, and housewares.

Interviewee: Does Big M Mart market its food products differently than do grocery stores?

Interviewer: Discount stores advertise lower prices for a wide variety of foods,
particularly staple, nonperishable foods.

Interviewee: Could I take a moment to write a few notes to myself?

Interviewer: Please feel free.

Set up the framework

Interviewee: Before making recommendations, I think we would need to evaluate


whether sales growth at Big M Mart is good or bad for Foods, Inc. To do that, I would
first look at how its sugar cereal performance at Big M Mart compares with that in
other distribution channels. Second, I would look at its performance at Big M Mart in
relation to competitors' performance. Next, I would determine what drives customer
purchases. Finally, I would want to understand the supply chain.

Interviewer: That certainly sounds like a reasonable approach. Let's proceed.

Evaluate the case using the framework

Interviewee: First, I would like to get a better sense of where Big M Mart stands in relation
to our client's other distribution channels by examining the client's sales data and margins, by
distributor.

48
Practice Cases – Case 6: Sugar Cereal BCG
Interviewer: The marketing department does not have margins by channel, but
tracks sales and volume for its top five distributors.

Sales ($M) 1997 1999 2001 5-Yr CAGR


Big M Mart 142 162 246 14.7%
R.J.'s 157 185 200 6.2%
Bozo Mart 143 175 189 7.3%
Ace Grocery 101 109 153 11.0%
Shoppers Mart 57 62 67 4.0%
Total Top 5 600 693 856 9.3%
Total All Distributors 1,000 1,079 1,150 3.6%

Volume (M boxes) 1997 1999 2001 5-Yr CAGR


Big M Mart 65 74 113 14.7%
R.J.'s 72 81 85 4.2%
Bozo Mart 65 77 80 5.2%
Ace Grocery 46 47 64 8.8%
Shoppers Mart 26 27 28 2.0%
Total Top 5 274 307 370 7.8%
Total All Distributors 450 468 487 2.0%

What does this imply about Big M Mart as a distribution outlet?

Interviewee: It looks as if the top distributors have been growing more important, but
particularly Big M Mart, which is growing faster than all the others. This is particularly true
when we look at volume, where Big M Mart's growth is much higher than that of the other four
channels.

Interviewer: And how could you interpret what these data says about margins?

Interviewee: While the client's sales through other distribution channels are growing faster
than volume, Big M Mart volume and sales growth are the same, so the average price paid by
Big M Mart has remained constant. That implies that sales growth at Big M Mart could have
negative implications for our client's margins.

Next, I would like to look at how our client is doing in relation to the competition within Big M
Mart. Have they been gaining or losing market share?

Interviewer: How might you find that out?

Interviewee: I would try to interview Big M Mart's purchasing personnel, since they would
probably track those data for their own purposes.

Interviewer: Why would they want to talk to you? How might you approach such an
interview?

49
Practice Cases – Case 6: Sugar Cereal BCG
Interviewee: I would approach the purchasing personnel and suggest that our client and Big
M Mart work together to identify best practices to reduce costs and increase sales of sugar
cereals at Big M Mart.

Interviewer: Let's say in a perfect world you could get a breakdown of Big M Mart
sales for the four largest competitors (see market shares below).

What can we infer about our client's competitors within this channel? Who should
they be worried about?

Interviewee: It looks like our client is losing market share, as is Tasty Breakfast, while
Cereal Co. and Private Label are gaining share. Private Label, however, looks to be growing
from a very small base.

I would like to explore why our client is losing market share to Cereal Co. at Big M Marts. Are
their prices better than those of our client?

Interviewer: After a period of price wars six to seven years ago that lowered
industry margins, the cereal companies have refrained from price competition within
the same channel.

Interviewee: If prices are not driving the difference, I would look at other factors such as
brand selection, percentage of shelf space, product placement, and in-store promotions.

Interviewer: Visits to Big M Marts indicate that each name-brand company holds 30
percent of the shelf space, while private label has 10 percent. Cereal Co. brands,
however, tend to be placed lower on the shelf than your client's products.

Interviewee: Well, I suspect that children are a large target market for the sugar cereal
manufacturers. The lower shelf placement could be especially important to children who are
looking at the different types of cereals. Are there any other promotions?

Interviewer: Some Cereal Co. brands have sales promotion tags, and the team notes
that store flyers advertise specials on Cereal Co. brands for Big M Mart customer
cardholders.

Interviewee: So, even if all the companies are maintaining product prices, maybe Cereal Co.
is strategically discounting prices to gain market share.

It seems as if there is evidence of cooperation between Cereal Co. and Big M Mart. Do we
know anything about their relationship?

50
Practice Cases – Case 16: PCB Manufacturer Booz Allen Hamilton
• Manufacturing:
What are the different costs in manufacturing? Neglect fixed costs for now (Plant,
property, equipment, Long-term agreements etc.)

We shall focus on variable costs.


o Direct Material Cost (it’s the cost of raw material)
o Labor Cost - $20/hr for US; $4/hr for China; $10/hr for Philippines
o Employee Productivity / efficiency
• Productivity: 80% for China; 90% for US and Philippines
• Efficiency: 75% for China; 90% for US; 80% for Philippines
o Overhead burden – Assume $10/actual labor hour for all countries
o Scrap rate / rework rate – cost of quality
o Tax rate

It takes 6 minutes to produce one unit in all three plants.

• Warehousing:
Why would you need warehousing cost?

For long-distance transportation, the client would need to carry more inventory on its
shelf to compensate for the long lead-time and demand variability during the longer
lead-time. It is important to realize safety stock needed to maintain certain service
level and reduce stock out costs.

Warehousing costs are 25% of sales in China and Philippines and 10% of sales in the
US.

• Transportation/Distribution:
The transportation cost would be higher when shipping products from country to
country.

Annual transportation costs are: $4M in China; $1.5M in US and $3M in Philippines

• Docking Costs:

These costs include customs, tariffs, inbound taxation rates etc.

• Marketing/Sales:
Ignore in this case

• Customer Service:
Ignore in this case

Interviewer: Let interviewee calculate the unit profit margin in each plant

Variable Cost China US Philippine

Raw Material $1/unit $2/unit $1/unit

Direct Labor (DL) $4/hr $20/Hr $8/Hr

Theoretical Time 6 min/unit 6 min/unit 6 min/unit

Productivity 0.8 0.9 0.9

Efficiency 0.75 0.9 0.8

94
Practice Cases – Case 16: PCB Manufacturer Booz Allen Hamilton
Scrap NA NA NA

Overhead $10/Actual DL $10/Actual DL $10/Actual DL


Hour Hour Hour
Warehousing 20% of sales 10% of sales 20% of sales

Transportation 4,000,000 1,500,000 3,000,000

Here are the tabulated results:

Variable Cost China US Philippines

Raw Material 6,000,000 12,000,000 6,000,000

Theoretical Rate 10/Hr 10/Hr 10/Hr

Actual Rate
(Adjust for efficiency
and productivity) 6/hr 8/hr 7/hr
Total Hours 1,000,000 750,000 900,000

Direct Labor Rate 4,000,000 15,000,000 7,200,000

Overhead 5,000,000 7,500,000 9,000,000

Warehousing 12,000,000 6,000,000 12,000,000

Transportation 4,000,000 1,500,000 3,000,000

Total Cost 34,000,000 42,000,000 37,200,000

Cost / Unit $5.2 $7 $6.2


Price/Unit
10 10 10

Margin $4.8 $3 $3.8

Conclusion:
The plant in China has a higher profit margin due to lower cost structure and therefore
expanding capacity in that plant will bring the highest return.

Interviewer: If we decide to invest in a brand new facility in China and move all
manufacturing from the US plant to China as well, what are the things we should
consider?

Some of the things to consider:


• US facility shutdown costs (severance, plant closures, machine sell-off etc.)
• Union issue
• Facility startup cost in China
• Additional costs to transport products back to US, such as transportation, customs, etc
• Currency risk
• Chinese government regulations
• Impact on brand image in US and US customer relations

Interviewer: Suppose the client decides to move manufacturing to China, what


would the client need to consider when deciding whether to invest in new
equipment?

95
Practice Cases – Case 16: PCB Manufacturer Booz Allen Hamilton
Sample answer:
• If the equipment has high value, and a specific asset to the client, the client should
consider shipping it overseas vs. selling off in the US and purchasing in China. Of
course, the client needs to compare the cost of both options. Leasing in China is
another option.
• The equipment needs to have a physical useful life for the ‘overhead’ allocation to be
low.

Interviewer: The client believes that the asset is specific. It is fully depreciated, and
has useful life of 10 years. Hence, the client decides to ship the equipment to China.
Uninstalling and shipping and then reinstalling will take about 2 months. How should
the client do in the interim to keep the business going?

Sample answer:
The client should focus on building a ‘bridge’ inventory in order to satisfy the customer
demand during the timeline between uninstalling and shipping the machine to China; and
installing it and running the machine to production.

96
Practice Cases – Case 17: Electronics Meter Manufacturer A.T. Kearney
(Source: Interview case from A.T. Kearney Round I, 2006)

Context
Your client is an electronics meter manufacturer with $200M revenue in the States.
They would like to enhance their profitability in five years and they need your advice.

A good framework may address the following areas:

• Have a better understanding of the business the client is in:


o Market performance - electronics meter market size and growth
o Product segments
o Customers
o Competitive landscape

• To improve profitability, the client would need to:


o Increase revenue
ƒ Increase price
ƒ Increase volume by stealing share, or stimulating demand
ƒ Expand into other markets
o Reduce costs
ƒ Assess the cost structure and/or value chain
o Optimize product mix to sell more products with higher profit margin

Given upon request

♦ The client has only one type of customer for various product segments: utility (electricity
only) companies
♦ Utility sector is highly regulated; # of utility companies is stable over the years
♦ Utility company buys the meters and installs them in apartments and buildings to
keep track of electricity usage
♦ Three product segments (our client is in “Centralized”):
♦ Walk-in – utility company sends teams to drive around apartments and buildings
and receive meter reading using RF technology
♦ Centralized – utility company needs to build infrastructure to connect with the
meters but does not need to send labor for meter reading
♦ Manual – utility company sends teams to enter apartments and buildings to read
meters
♦ The market situation of these segments:

Segment % of Market No. of Players Tech Requirement Growth


Walk-in 50% 15 Medium Moderate
Centralized 30% 2 (including High Fast
our client)
Manual 20% 10 Low Slow

Interviewee should realize our client is in an attractive segment - “Centralized” because of


higher entry barrier, fewer competitors and fast growth, although it is not the biggest.

Interviewee should then explore opportunities to increase revenue.

Given upon request


• Current price per meter: $30 (walk-in) vs. $50 (centralized); no information on price
sensitivity.

Options to increase revenue:


• Increase price:
o There are only two companies in “Centralized” segment. However, increasing
price might lead to share loss to other segments because targeted customers
are the same.

97
Practice Cases – Case 17: Electronics Meter Manufacturer A.T. Kearney
• Decrease price to gain more volume in “Centralized”
(Ask interviewee what would be the expected cost level of each segment.)
o “Centralized” may have the highest cost structure because of high tech
requirement, followed by “Walk-in” and “Manual”. Based on the current price
and cost structure, our client is unlikely to compete with players in other
segments on price alone.
• Expand into “Walk-in” segments
(When interviewee realizes our client could expand into the second most attractive
segment – “Walk-in” (“Manual” segment seems too small and has poor growth), give
the following information)
o Our client has the technology to enter other segments, however, researching
and applying the licenses for “Walk-in” products will take about four years.
o Our client is too small to acquire or form a JV with a player in the “Walk-in”
segment.
o On the other hand, players in other segments cannot easily enter our
“Centralized” segment in the near future because of technology hurdle.
• Other expansion ideas
o Develop new customer segments (non-electricity)
o Expand into new markets, e.g. overseas
o Develop new products

Interviewer: All the other expansion ideas will take more than five years. What could
other players do even they cannot enter our client’s segment in the short run?

Sample answer:
Those companies could under-cut our price due to their lower cost structure and try to take
customer away from our client. However, we need to find out what our customers value the
most. They might not be price sensitive.

Interviewer: Ask interviewee what are utility companies’ decision factors.


Sample answer:
o Upfront investment: meter cost and infrastructure (for “Centralized” only)
o Ongoing cost: labor and maintenance
o Relationship: all players in the market have similar relationships with utility companies
o Customer service
o Switching cost: very hard to persuade utility companies to switch from one product
type to another due to previous investment; main source of growth is from meters for
new locations.

Interviewee should realize upfront investment for “Centralized” is higher but ongoing cost will
be lower because of labor cost saving.

Interviewer: Ongoing cost savings of “Centralized” can more than justify the higher
upfront investment, however, utility companies care more about upfront cost when
buying meters. Ask interviewee how our client can persuade utility companies to
buy “Centralized” meters.

Sample answer:
o Leasing the meters instead of selling
o Allowing customers to pay by installments

By now, interviewee should realize it is hard to increase revenue (on top of the segment
growth), and our client needs to decrease cost in order to improve profitability and
competitiveness.

98
Practice Cases – Case 17: Electronics Meter Manufacturer A.T. Kearney
Options to reduce cost

Given upon request


o No information on cost structure
o Currently, client has only one plant in the U.S.

Interviewee should do value chain analysis and brainstorm ideas to optimize value chain
components

Sample ideas:
o R&D – improve design to lower cost
o Raw material – use cheaper substitutes, negotiate with suppliers for better price (or
shop around for best suppliers), leverage economies of scale
o Manufacturing – deploy labor-saving technologies, make process more efficient,
improve utilization, outsource (brainstorm potential issues with interviewee),
relocating production to Mexico (brainstorm possible locations and potential issues
with interviewee; main idea is to minimize distribution cost increase and lower
production cost), JIT production
o Distribution – optimize warehouses and distribution network

Interviewer: Ask interviewee how to construct a DCF (Discounted Cash Flow)


analysis for plant relocation in Excel, ignoring tax. Brainstorm on Excel layout and
items.

Given upon request:


o Existing equipment will be moved to new location and existing production workers
would be terminated (brainstorm potential issues with interviewee).

Sample answer:
The simplest way is to come up with an incremental analysis, i.e. difference between the
current situation and the future one.

o Columns: Year 0 to 5 (example)


o Main items:
o Cost savings (ask interviewee how to calculate) – # of units * (labor $ saving
per unit – extra distribution $ per unit)
o Initial investment (year 0) – land (if necessary), relocation of equipment,
compensation to plant workers in the US
o Working Capital (WC) Requirement and Change in WC
o Calculation: assume certain growth rate for unit production in each of the first five
years and calculate the terminal value for year six onwards based on perpetual growth
rate; discount each year’s cash flow to year 0 using cost of capital (ask the
interviewee what it is).

Interviewer: Ask interviewee to describe the definition of WC and its main source.
Sample answer:
o Definition: Current Asset – Current Liability
o WC is positive because companies normally need to pay their suppliers before they
receive money from customers.
o Main source of WC is inventory.

Interviewer: Ask interviewee why our client may require more WC in their new plant.
Sample answer:
o They might need more safety stock due to:
o Potential manufacturing issues due to new labor and plant
o Possible distribution issues – longer distance and hence higher chance of
delivery delay

99
Practice Cases – Case 17: Electronics Meter Manufacturer A.T. Kearney
Interviewer: The DCF analysis of plant relocation has a positive NPV. Ask
interviewee to make a recommendation.

Main points of recommendation:


♦ Limited opportunity to increase revenue beyond “Centralized” segment growth
♦ Cost cutting is necessary to improve profitability, for example, plant relocation
♦ Risks and issues associated with plant relocation and other cost cutting measures, e.g.
labor and regulation issues of relocating plant, intellectual property issues with
outsourcing
♦ Next steps

100
Practice Cases – Case 19: Direct Mail Campaign Deloitte.
(Source: Interview mini case from Deloitte Round II, RSB 2006 Casebook Case#19)

Context
Our client is a telecom fortune 50 company. One of its divisions (landline) is facing
high churn (3% per yr). They have launched a direct mail campaign to try to re-
attain some customers. How can they make their direct mail strategy more effective?
They have kept all the information they had on our customers.

A sample approach interviewee could use:

To make the direct mail strategy more effective, I’d like to focus on three areas.

First I’d like to have a better understanding of our customers so we could focus the campaign
on the right customer segment. Second, I want to make sure that the content of the direct
mail is effective. Third, I want to make sure that the channel we use to deliver the campaign is
effective.

To understand our customers better, first I’d like to know why customers switched to other
providers. It could be because our competitors offer lower price, or better quality, or better
customer services, or more selections.

Interviewer: There is no significant difference in the quality of service provided. It is


standard landline service. The client also provides other services such as broadband,
and wireless to meet diversified customer needs. Therefore, the main reason for
customers to switch is price.

(Note: Interviewer should guide interviewee to discuss customer segmentation.)

Interviewer: What are the different ways to segment customers?

A good answer may include the following:


- Customers can be segmented by different needs. For example, business customers
might have different needs from individual consumers. Within the business
segment, large enterprise customers might have different needs from
small/medium business owners.
- Customers can be segmented by life-time economic value by looking at revenue
generated by customer, costs incurred to serve that customer and the number of
years the customer stays with us. Since we have kept all the information about
each customer, it shouldn’t be hard to calculate customer economic value.

The direct mail campaign should target at customers who have had high life-time economic
value or have potential to bring in high economic value.

The content of the campaign should focus on what customers value the most and target at the
reason why customers left in the first place. Since customers leave mainly because of price,
the campaign can focus on current promotions and discounts.

It’s also important to effectively utilize distribution channels. For example, we could use higher
cost channel such as standard US mail to reach customers with higher profitability, and use
lower cost channel such as internet to reach customers with lower profitability.

(One important point is to realize that the cost of serving every customer is not the same.
Costs of serving customer depend on frequency of customer calling the service center, how
often bills are not paid on time; etc. The client can use the data to compute cost of serving the
customer. Hence, it’s important to segment customers based on “profitability” and not
revenue.)

103
Practice Cases – Case 20: Cleaning Supplies Manufacturer ZS Associates
(Source: Interview case from ZS Associates Round I, RSB 2006 Casebook Case#20)

Context
Your client is a Cleaning Supplies Manufacturer. They are trying to redesign their
Sales Force and have approached you for advice.

A good clarifying question to ask in the beginning:


What is the reason behind the redesign initiative? Is it because the sales force is not meeting
sales goals or because of some other reasons?

Interviewer: Good question. The client has found out that its sales force performance is below
industry average and wants to improve sales force effectiveness through the redesign.

Interviewee should first structure the analysis. A sample approach:

First, I’d like to find out how the sales force is currently structured. Does each salesperson sell
multiple products or each salesperson is specialized in selling certain products?

Then, I’d like to assess sales force performance by product category to identify areas to focus
on first. I would use metrics such as annual sales per salesperson to assess the performance.

Third, I’d like to understand the selling process such as how sales people reach customers and
how much time is spent on what tasks to identify where the inefficiency exists. We could do a
benchmark analysis if data is available. I will also look into the resources/tools used by sales
force to see if resource allocation might have affected sales force effectiveness. It’s also
important to check the incentive system to see how sales people are compensated. We need
to make sure that the incentive system is in line with the company’s financial objectives.

Once I identify the problem, I will explore ways to increase the sales force effectiveness and
analyze possible impact and risks should we implement any changes.

Interviewer: show the following exhibit upon request and ask what products the
client should focus on first to improve sales.

Four product lines A, B, C, D

# Current Prior Year


Customers Revenue Revenue %Change
A 12,000 $24M $18M 33%
B 24,000 $30M $35M -14%
C 6,000 $24M $20M 20%
D 3,000 $30M $18.5M 62%

A good answer would lay down a few scenarios:

There are a few ways we can use to determine priority.


• If the client is concerned with declining sales, we should start with B because its
revenue has dropped.
• If the client is concerned with maximizing revenue per customer, then we should focus
on products that have lowest revenue per customer. However, the comparison in such
a way might not make sense because products are different and customer segments
are different.
• If the client is concerned with revenue performance relative to competitors, then we
can benchmark sales data against competition if data available and focus on the ones
with the worst revenue performance.

104
Practice Cases – Case 20: Cleaning Supplies Manufacturer ZS Associates
• If we have data on sales force, we can find out how much sales each salesperson
brought in by product and focus on products with the lowest sales force effectiveness.

Interviewer: Suppose a sales person can make 3,000 calls per year, and on average
a customer must be called 10 times to make a sale. How many sales people should
you hire?

Answer: Assuming there is no overlap in customers of each product, there are 45,000
customers in total. If we want to make a sale from each customer, we need to make 450,000
calls and that means we need 450,000/3000 = 1,500 sales persons.

Interviewer: Show the following exhibit. “The Research Team has provided us the
following revenues and costs data associated with the sales force. What would be
the optimal number of sales people by eyeballing it?”

Answer: The optimal number of sales people is the number that maximizes the profit, i.e. the
biggest gap between the Revenue curve and the Cost curve if a vertical line is drawn.

Interviewer: Using the same slide, if you are given the following:
Revenue = 3,000,000*sqrt(N) + 10,000,000 (N = number of Sales People)
Gross Margin = 80%
Fixed Costs = 8,000,000
Assume each sales person makes $100,000 in salary and benefits

Now what’s the optimal number of sales people?

Answer:
Profit = 80%*(3,000,000*sqrt(N)+10,000,000)-100,000N-8,000,000
= 2,400,000*sqrt(N)-100,000N
To maximize profit, first derivative should be zero and second derivative should be negative.
• First Derivative: 1,200,000/sqrt(N)-100,000 = 0 Æ sqrt(N) = 12 Æ N = 144
• Second Derivative: -600,000/sqrt(N^3) Æ Negative

55
50 Revenue
45 Costs
40
Millions of $

35
30
25
20
15
10
50 100 150 200
Number of Sales People

105
Practice Cases – Case 21: Trucking Company Diamond
(Source: Interview case from DiamondCluster Round I, RSB 2005 Casebook Case#25)

Context
Your client is a trucking company. The company has grown through acquiring
regional trucking companies that are currently managed as separate businesses.

The CEO, who is new and an outsider to the company, is interested in a Route
Optimization software that has been developed by one regional subsidiary (Region X)
and has significantly improved profitability within the subsidiary. The CEO asked
you to see if the software should be implemented companywide. The CEO also wants
your advice on where the company should go in the long term.

Interviewee should ask questions to learn more about the software.

Given upon request

The software is used to optimize routes within a region to make truck pick-ups and drop-offs
more efficient. The software doesn’t optimize long haul routes.

(A, B, C are locations for pick-ups and drop-offs)

Region 1 Region 2

A D

Long Haul
B E

C F

A sample approach to structure the analysis:

To assess if we should roll out the software to other regions, I first would like to understand
how the software improves profitability. I want to understand what benefits the software
offers and the impact it has on both revenue and cost. On the revenue side, maybe the
software enabled the client to provide better and faster customer services and therefore
customers use our services more often or more customers are acquired. On the cost side,
there might be some cost savings opportunity due to route optimization.

Then I’d like to estimate what would be the incremental revenue and cost if we roll out the
software companywide. In the meantime, I’d also like to assess the non-financial impact the
companywide roll-out might have.

At last, I will discuss some of the long term strategies the CEO can look into.
Interviewer: Ignore revenue and focus on cost. Ask interviewee to identify cost
buckets of the trucking companies

106
Practice Cases – Case 21: Trucking Company Diamond
Examples of the cost buckets:

• Fixed cost
o Real estate
o Equipments including trucks
o Insurance
o Overhead
• Variable cost
o Direct labor (trucker’s salary etc)
o Fuel

Given upon request

• Current pick-ups or drop-offs/ hr: 2. Pick-ups or drop-offs/ hr with the software: 2.5
• Truckers are paid at an hourly rate of $100
• Companywide annual pick-ups are 4M, drop-offs are 4M
• Region X annual pick-ups are 0.8M, drop-offs are 0.8M
• Labor is 1/3 of total costs
• Labor is split 50/50 between pick-ups & drop-offs
• Set up cost to expand the software companywide: $20M.
• Incremental annual cost to maintain and support the software: $15M

Ask interviewee to calculate the incremental cost savings if implemented companywide.

Before using the software, annual labor cost is:


• Region X: ((0.8M+0.8M)/2) * $100 = $80M
• Companywide: ((4M+4M)/2) * $100 = $400M

After using the software, annual labor cost is:


• Region X: ((0.8M+0.8M)/2.5) * $100 = $64M
• Companywide: ((4M+4M)/2.5) * $100 = $320M

Annual cost savings:


• Region X: $80M - $64M = $16M
• Companywide: $400M - $320M = $80M
• Incremental cost savings if implemented companywide (Region X already realized the
cost savings): $80M - $16M = $64M

If the software is implemented companywide, incremental cost savings are $64M, incremental
costs are: $35M in year 1 and $15M yearly after. Therefore, it makes financial sense to
implement the software companywide.

A great answer will also address the non-financial impact of expanding the software
companywide on the current IT system and operation, and discuss the possible benefits of
enhancing the software to optimize long haul routes.

After assessing the investment in expanding the software, interviewee should lay out a
structure to discuss areas the CEO could look into for the long term.

Examples of areas to look into:


• Increase revenue
o Optimize service portfolio. For example, acquired trucking companies can work
together to provide more long haul services
o Enter new markets (new geographical locations, new customer segments, etc.)
• Decrease cost
o Consolidate common business functions, such as Accounting, HR
o Consolidate procurement to increase bargaining power
o Standardize certain processes

107
Case 5: Syzygy Supercomputers
By: Edwin Van Dusen, Brian Fox and David Welch (Kellogg Class of ‘04), Edited By: Uri Kalir (Kellogg Class of ‘12)

Case Question

 Syzygy Supercomputers is a large international fully-integrated computers and communications company with annual
revenues of approximately $20 billion U.S.. In the past several years, the company has seen a steady decline in profits.
3
 The CEO has asked us to look into this problem. How can Syzygy Supercomputers get back on track? Quants.

7
Structure
Case tracker Fit Questions Guide to interviewer

 Industry: Spend first 15 min on fit  This case is about a tech company undergoing a decline
Tech & Telecom in profitability, despite a strong competitive position.
 What is the single most Profit Imp.
Syzygy has been overinvesting in R&D relative to its
 Level of Difficulty: important detail in your Comp. Anl
returns and performance has suffered as a result.
Hard resume? Ops.
 The interviewee is expected to go through these general
 Case format:  What is the least steps while presenting a solution:
Improving profitability important detail in your
1. Confirm profitability issues and ask to see
resume?
 Concepts being tested: historical revenue and cost information.
 Why is the lifestyle of 2. Ask to see market position and related data.
- Competitive
this job right for you? 3. Realize that this case is going to be decided
analysis
based on information regarding the products.
- Operations 4. Identify the main issue affecting profitability –
the company’s R&D spending

© 2011 Kellogg Consulting Club -- All Rights Reserved 42


Clarifying answers and case guide

Clarifying answers to provide if


Interviewer Guide to Case
Asked
General Information on Syzygy A sample case structure would include the following:
- Only the information that has already 1) Examination of historical performance to deep-dive declining profits.
been given. Other information currently 2) Discussion of market dynamics and competitive positioning.
unavailable. 3) Deep-dive into drivers and profitability , ultimately leading to a discussion of
Competitive Dynamics SG&A
- There are several players in the market, Necessary Information that should be given only when specifically asked for by
but everything has remained stable interviewee:
from a competitive standpoint.
 Historical costs, revenues and profits – exhibit 1
Profitability drivers  Note that the interviewee should be asking for historical trends in the
company’s profitability. If they are asking only for current profitability,
- COGS, SG&A, and other profitability
interviewer should point them in the direction of historical trends
factors are on-par with Syzygy’s
competitors (except for R&D, about  Market data – exhibit 2 followed by exhibit 3
which the candidate should ask  Interviewer should give the interviewee a few seconds to understand
specifically). slide 2 and then handout slide 3. Interviewer should ask interviewee
which product in slide 3 is which (on slide 2).
 R&D spending data – exhibit 4 and exhibit 5
 The candidate will naturally ask about potential drivers of profitability
(S&GA expense, COGS, etc.).
 These exhibits should only be given once the candidate has identified
R&D as a driver of Syzygy’s profitability.

© 2011 Kellogg Consulting Club -- All Rights Reserved 43


Key elements to analyze

Interpreting Exhibit 3 Notes on Exhibits 2 & 3 Notes on Exhibits 4 & 5


 The X axis represents absolute market  Correctly interpreting slides 2 and 3  The key insight in exhibit 4 is that
share (AMS), which correlates with will lead the candidate to correctly Syzygy is the only major competitor in
the height of Syzygy’s rectangles in identify the products on slide 3: this industry whose R&D spending is
exhibit 2. below the “normative band”
 Product #1 is custom
 The Y axis represents relative market applications  We would expect each company’s
share (RMS), calculated as Syzygy’s degree of technical leadership to go
market share divided by the market  Product #2 is supercomputers
up as it invests in R&D, but Syzygy,
share of the closest competitor. If  Product #3 is telecom with slightly more R&D spending than
Syzygy is the market leader, this equipment Cray Research and Sonic Wave, enjoys
number is greater than 1. If Syzygy is less of a perception of technical
 Product #4 is satellites
not the market leader, its market leadership from its customers
share divided by the market leader’s  Product #5 is operating
share will be lower than 1. This software  Exhibit 5 builds on this point. The key
correlates with the the ratio of the insight is the absolute expense (not
height of Syzygy’s rectangles to that of percentage) Syzygy is spending on
competitors’ rectangles for each R&D. Correct analysis will show that
product in exhibit 2. Syzygy is spending money on features
that do not generate customers’
 The diameter of the bubbles is driven “willingness to pay”. Syzygy is
by Syzygy’s total revenues in that spending money to develop features
category, comprised of its market that consumers are not willing to pay
share in the category multiplied by the for.
category’s size.

© 2011 Kellogg Consulting Club -- All Rights Reserved 44


Solution and recommendations

Solution & Recommendations


 The candidate should be able to produce one of the following insights:
 Cut R&D to eliminate spending on unnecessary customer features.
 Keep R&D spending the same, but switch the work to developing features that consumers will be willing to
pay for at a level that is higher than the R&D expense itself.
 A second-order insight from the case is that Syzygy should exit the operating software business (product #5 in this
case) since it’s small market share will continue to erode as a result of economies of scale, network economics and
the experience curve all working against it.

Bonus/Guide to an Excellent Case


 An excellent interviewee will –
 Take a few seconds to fully understand each of the complex graphs he/she is given before giving an answer.
 Immediately ask about R&D spending and not go through COGS or SG&A first (this is a hi-tech company, so
R&D spending should be looked-at before other drivers of profitability).
 Conclude their recommendation by not only stating that Syzygy should exit the operating software business,
but also focus on the categories in which it is the market leader in order to get some or all of those
economies to work for it and against its competition.

© 2011 Kellogg Consulting Club -- All Rights Reserved 45


Exhibit 1: Syzygy’s Historical Costs, Revenues and Profits

© 2011 Kellogg Consulting Club -- All Rights Reserved 46


Exhibit 2: Worldwide Telecom Computing Market (Today)

© 2011 Kellogg Consulting Club -- All Rights Reserved 47


Clarifying answers and case guide

Clarifying answers to provide Guide to case / Guide to handouts


Client Characteristics Exhibit 1– Provide once interviewee receives clarifying information
- Vitality is the leader in its category and (left pane) and asks for more detail on costs
has over 10K field sales agents
Interviewee should recognize the following:
- Vitality sells all policies through its field
sales agents who are solely compensated - All line items except for sales costs growing at 10% per year
on a % commission of total new premium, - Sales costs grow at 10% from 2008 to 2009, but at 45% from 2009
defined as premium from new customers to 2010 (while premium growth remains at 10%)
or additional premium (up-sell) from - Stronger interviewees will quickly note that something is strange
existing policyholders
w/the 2010 sales costs, but will calculate to confirm
- In addition to the commission, short term
priorities are often communicated via - Finally, profit margins are declining significantly from 09-10,
sales contests that focus on a particular suggesting that the increase in sales costs is not paying off
customer segment or activity Exhibit 2– Provide if interviewee asks about the value chain or selling
- Major costs: sales, G&A, and advertising process. If the interviewee asks about the new contests focused
Competition on premium, provide the following information:
- Vitality has a few other competitors in this
market who have seen similar growth, but - Vitality launched a contest called “Sweeps Week” that aimed to
Vitality is a leader in the space and thus drive increased premium in weeks that were traditionally low
competition is not the focus volume for the company. Vitality paid an extra 10% commission
Industry trends on all premium booked in those two weeks.
- Mature market - Sales agents thought “Sweeps Week” was a great contest
- Agent turnover is very high on a yearly
- We have no info on the additional two contests on new accounts
basis (though was lower during the
recessionary period) Exhibit 3- Provide if interviewee asks for further detail on the
effectiveness of “Sweeps Weeks”

© 2011 Kellogg Consulting Club -- All Rights Reserved 123


Key elements to analyze

Supply/value chain Marketing strategy


 As seen in Exhibit 2; Vitality’s sales agents are engaged  Exhibit 3 shows weekly premiums for all of Vitality from 2008
in several activities along the sales process, and that – 2010. The chart is shown in a way that compares each
new premium can be generated in many ways. year’s actual premium to the average of historical premium
for that year, so as to provide for a “benchmark” comparison.
 The interviewee should recognize the shift in contest
mix from 2008/09 to 2010. Coupled with the additional  The “Sweeps Week” contest launched in 2010 is shown in
information provided in the previous slide, the weeks 4 and 20 in the chart, and clearly yields high premium
interviewee should realize that Vitality has shifted its volume for those particular weeks. However, it is done at the
focus more toward total premium and new accounts in expense of the weeks surrounding the sweeps week.
2010  This implies that agents may be “gaming” the system by
pushing/pulling sales into that week to earn the contest $$

Notes to interviewer Notes to interviewer


 Assume the types of contests run in 2008 and 2009 were  If asked, confirm that agents have authority to “book” sales
fairly similar whenever they want by influencing the enrolment timing by
up to one week
 Do not share Exhibit 3 until the interviewee recognizes
this shift in mix and begins asking questions about the  Strong interviewees will also recognize that, despite not
new programs having charts to support it, a shift in customer focus toward
acquiring new accounts in 2010 will likely also hinder profit
margins, as acquiring a new customer costs considerably
more than retaining (or up-selling) an existing customer

© 2011 Kellogg Consulting Club -- All Rights Reserved 124


Solution and recommendations

Solution & Recommendations


 The interviewee should conclude that Vitality overspent in 2010 on the “Sweeps Week” sales contest, thus hurting
its profitability
 Exhibit three indicates that the contest influenced the sales force to conduct undesirable selling practices by
pushing / pulling forward business to earn the extra commission
 Further, the contest’s focus, driving new premium was duplicative with that of the main commission system,
therefore it did not add much value
 Recommendation: eliminate “Sweeps Week” for 2011 and potentially repurpose those funds toward an activity that
is not already covered by the main commission structure
 It is unclear whether we should remove the increased contests on new accounts, as this could be a new customer
focus that we do not know about

Bonus/Guide to an Excellent Case


 Strong candidates will make the following observations:
 Recognize that the likely decline in profit margin from 2009 to 2010 is linked to the abnormal increase to
sales costs
 That the “contest mix” in Exhibit two is similar to a firm’s marketing mix, especially given that the sales
channel has already been established as the main marketing channel for these products
 Suggest that the added contests on acquiring new accounts will also decrease profitability because acquiring
new customers is more costly than retaining existing ones.

© 2011 Kellogg Consulting Club -- All Rights Reserved 125


Exhibit 1: Vitality insurance results and major costs

Vitality insurance key results and costs (Figures in 000s)


2008 2009 2010
Accounts converted 500 550 605
Total policyholders enrolled 1,500 1,650 1,815
Total premium from policyholders $2,500,000 $2,750,000 $3,025,000

Total costs
General and Administrative $50 $55 $58
Sales $250 $275 $400
Advertising $25 $28 $30

Profit margin 9.50% 9.40% 8.50%

© 2011 Kellogg Consulting Club -- All Rights Reserved 126


Exhibit 2: Vitality insurance sales process

Reactivating
dormant
account

Keep Gain active Produce


Hire new Acquire new results
accounts policy
agents accounts
active holders (Premium)

# of agent focused sales contests targeted at these leverage points on the selling process

2008 N/A 2 4 1 2
Year

2009 N/A 2 4 1 2
2010 N/A 4 0 1 4

© 2011 Kellogg Consulting Club -- All Rights Reserved 127


Exhibit 3: Snapshot of “Sweeps Week” contest results

Ratio to Average Weekly Premium (2008-2010)


200%

2010 “Sweeps 2010 “Sweeps


Week” (1) Week” (2)
150%

Yr 2008
Yr 2009
100%
Yr 2010

50%

0%
1 2 3 4 5 6 7 8 9 10 11 12 13 14 15 16 17 18 19 20 21 22 23 24 25 26 27
Week during Jan-Jun

© 2011 Kellogg Consulting Club -- All Rights Reserved 128


Case 19: Chic Cosmetology University
By: Peter Manoogian (Kellogg Class of ‘12)

Case Question

 Our client is a for-profit, specialty college named Chic Cosmetology University (CCU). Founded in 2005, CCU is a program
for high school graduates seeking their professional cosmetology license. CCU is currently the market leader for
cosmetology education with campuses in ten major metropolitan areas in the US. 7
 CCU has capital to invest in a new campus and is considering Chicagoland as a location – should they do it? Quants.

 (If interviewee asks about OTHER objectives or defining success for opening the new location): The client considers a 8
successful launch as achieving positive operating profit for the new campus two years after opening
Structure

Case tracker Fit Questions Guide to interviewer


 Industry: Spend first 15 min on fit  This case involves some number crunching but is more
Education structurally focused. It is critical to ensure that the B/E
 Tell me about a difficult
 Level of Difficulty: or sensitive situation interviewee lands on the figures presented (or is course Mkt. Stgy
Medium corrected toward them) in order to proceed with the Mkt Share
that required careful Invest.
 Case Format: later parts of the case.
communication
Opportunity  The interviewee should be able to develop a variant of
Assessment  Describe a project or the following question:
idea that was introduced Will CCU be able to enroll enough students to offset
 Concepts Tested: or implemented because the initial investment and achieve positive profit?
- Break-Even Analysis of your efforts  Key case steps:
- Marketing Strategy 1. Evaluate CCU revenue and cost structures
- Market Share 2. Project CCU’s market share
- Investments 3. Estimate CCU Chicago enrollments
4. Identify qualitative issues to consider

© 2011 Kellogg Consulting Club -- All Rights Reserved 129


Clarifying answers and case guide

Clarifying answers to provide Guide to case / Guide to handouts


Client Characteristics Share Exhibit 1 with interviewee after probing questions are
- Enrolled students take classes at a received about CCUs revenues and costs . Interviewee should be
physical campus for one school year to able to compute the following:
earn degree (FY begins on 9/1)
- Average revenue per enrollment = $15K (revenue / total
- CCU boasts the best campuses in the
industry with state of the art equipment enrollments)
- Strong job placement due to CCU’s - Total annual fixed cost per campus = $4.8M ($48M / 10)
relationships with top salons in local areas - Total variable cost per student = $8K ($80M / 10K)
- CCU and industry enrollments growing at
- Gross profit per student = $7K ($15K - $8K)
5% per year
Competitive Dynamics After that, the interviewee should begin to tackle the overall
- 2-3 other large specialty colleges, some of opportunity in the area as well as how many enrollments CCU
which are in the same geographies as CCU could reasonably expect to obtain in year 1
- Community colleges beginning to offer
cosmetology degrees at lower prices Share Exhibit 2 with interviewee after some of the qualitative
- All ten CCU campuses have been present aspects of CCU’s targeting and marketing strategy are covered.
for at least three full school years Additionally, the interviewee should have asked about competitor
- All competitor campuses have also been information or made some attempt to assess what share of the
present for similar lengths of time market they should expect in Chicago.
Industry Characteristics/Economics
- See “Market Share” section in next slide for further
- H.S. Diploma and cosmetology degree
required to enter the field
information on Exhibit 2
- 98% of cosmetologists are women

© 2011 Kellogg Consulting Club -- All Rights Reserved 130


Key elements to analyze

Break-even analysis Marketing strategy Market share


 How many students will CCU need  What types of schools / students do you think  What is the highest share we could
to break-even in year 1? CCU targets? expect CCU Chicago to capture in Y1?

Notes to interviewer Notes to interviewer Notes to interviewer


• Interviewee should ask about the Possible responses (schools): • Show Exhibit 2 to interviewee
investment cost of building a new -Public schools (private HS more likely to have • Interviewee should recognize:
campus in Chicagoland grads go to 4yr univ) - Campuses w/competitors present
• After asking, tell interviewee that -HS’s in middle-class cities (may be an tend to have a lower share (10%)
the total cost is $4.5M in initial affordability issue w/lo income) than those w/out (15%)
building costs to renovate its chosen
-Closest to the campus (geography) - However, presence of >1
site. These costs can be amortized
evenly over a three year period. Possible responses (students) competitor does not have an
• Assume fixed costs remain flat per -Women HS graduates increased negative impact on
year market share (Boston has 8%
-Not attending 4yr college
share w/only one competitor)
-Interested in cosmetology
Interviewee should calculate - Interviewee should assume a
• Total fixed costs per year = $6.3M After a few of the above criteria are noted,
share:
projected 10% market share for a
($4.8M + $1.5M from amortization). Chicago campus (one competitor)
• Gross profit per student = $7K - CCU has identified 1,000 targeted high
schools in the Chicagoland area • Also share the following :
• Break-even number of enrollments
80% of enrolled students directly from
per year = 900 ($63M / $7K) - Within these HS, CCU’s market research
high school, the other 20% of students
estimates that on average 6 students per HS
come from the “Adult” market
have “potential” for CCU enrollment

© 2011 Kellogg Consulting Club -- All Rights Reserved 131


Calculations

Math questions
1. What is the breakeven number of students required for CCU Chicago?
2. How many students should CCU Chicago expect to enroll in year 1, at the most?
3. (If time permits) = Suppose CCU finds a lot with a one time construction cost of $300K with the same
amortization schedule. Should they enter Chicago now?

Calculations
1. Break-even: (Estimated campus fixed cost + Yearly amortization ) / gross profit per student
( $4.8M + $1.5M ) / $7K = 900 students

2. Students: Total potential students x maximum projected market share


Total potential students = Potential (HS) + Potential (Adult) = 7,500 students
Potential (HS) = 6 / HS X 1,000 targeted HS = 6,000 students
Potential (Adult) = 6,000 students * 25% = 1,500 students
Total potential students (year 1) = 7,500 students x 10% share = 750 students
Total potential students (year 2) = 750 students x 1.05 (growth) = 788 students

3. Break-even: (Estimated campus fixed cost + new yearly amortization ) / gross profit per student
( $4.8M + $0.1M ) / $7K = 700 students

© 2011 Kellogg Consulting Club -- All Rights Reserved 132


Solution and recommendations

Solution & Recommendations


 Overall, our client should NOT enter the Chicago market under the current cost structure. Even with a 10% market
share assumption in year 1, the Chicago campus will enroll only 750 students, this is 150 fewer than the 900
required to break-even.
 The client should also consider several qualitative issues:
— Consider offering scholarships to increase the number of potential students and/or conversion rate of potential
students
— Consider other cities beyond Chicagoland that currently do not have a CCU presence, or add a second campus
to a city such as NYC that has high market share and potentially low capacity
— Perform market research in other cities to understand if there is a greater potential per target high school to
increase ROI
— Consider ways to reduce fixed costs (e.g., transporting equipment / materials from campuses that are not at
capacity)

Bonus/Guide to an Excellent Case


 Excellent interviewees will recognize that the 10% market share is for campuses that have been in place for at least
three years , therefore Chicago is unlikely to achieve that share in year 1 or 2; this rules out the feasibility of the
follow-up question that suggests the possibility of reducing the one-time investment from $4.5M to $300K.
 Additionally, a strong interviewee will identify several of the qualitative issues listed above as ways in which CCU
could proceed

© 2011 Kellogg Consulting Club -- All Rights Reserved 133


Exhibit 1: 2010 CCU Financials

CCU Financials as of 8/31/2010 ('000s)

Student enrollments (all campuses) 10


Revenue from enrollments $150,000

Total fixed campus costs $48,000


Buildings and equipment $32,000
Recruiting, general, and administrative $16,000

Total variable campus costs $80,000


Instructors $40,000
Student supplies $40,000

Operating profit $22,000

© 2011 Kellogg Consulting Club -- All Rights Reserved 134


Exhibit 2: CCU and major competitor campus locations

As of 8/31/2010
2010
Campus share*
San Fran 8%

LA 14%

Minneapolis 15%

Dallas 14%

Houston 10%

Atlanta 16%

Miami 12%

Philly 12%

New York City 16%

Boston 8%

CCU Campus
Major competitor campus
* Measured as share of total “potential” students, as defined by CCU
© 2011 Kellogg Consulting Club -- All Rights Reserved 135
Case 20: DigiBooks Inc.
By: Shobhit Chugh (Kellogg Class of ’11), Edited By: Adam Louras (Kellogg Class of ’11)

Case Question

 Our client, DigiBooks, is a manufacturer and seller of electronic book readers . DigiBooks also distributes e-books for the
e-reader through their website. The reader is only compatible with books sold through the DigiBooks site.
 DigiBooks is planning the launch of its e-book readers in a country where no e-book readers are currently sold. Only 1% of
the population has ever used a e-book reader, though 50% is aware of the concept. The Chief Marketing Officer of 4
DigiBooks has come to you to help determine: Quants.

 How should DigiBooks launch and market the e-reader product in this new country? 7
Structure
Case tracker Fit Questions Guide to interviewer

 Industry: Spend first 15 min on fit  The case primarily tests the understanding of marketing
Tech/Telecom concepts, specifically a new product launch.
 Tell me about a time MKT Stgy
 Level of Difficulty: when you failed. Cust. Stgy
 The case is written in McKinsey style format; the Creativity
Easy
 Share me a time when interviewer is expected to guide the interviewer step by
 Case Format: you faced a difficult step through each question.
Developing a new situation in a team and
product how you solved this .  Begin by laying out the situation and case question, allow
 Concepts Tested: the interviewee to layout their structure, and then jump
immediately to question 1. The interviewer is expected
- Marketing Strategy to drive this case rather than the interviewee.
- Customer strategy
- Creativity

© 2011 Kellogg Consulting Club -- All Rights Reserved 136


Clarifying answers and case guide

Clarifying answers to provide if


Interviewer Guide to case and handouts
Asked
Industry Definitions Case Structure – Interviewee’s structure should cover the key areas needed to explore in
order to determine how DigiBooks should launch and market the reader in this country.
- E-book reader: Is a software, hardware
• The interviewee should take a few minutes to sketch out a framework for
and network platform that utilizes
analysis of the marketing plan
wireless connectivity to enable users to
shop for, download, browse, and read e- • Key elements expected to be included in this framework are:
books, newspapers, magazines, blogs, • Segmentation, targeting and positioning: Are there particular segments
and other digital media. in the population that will be ideal customers for us?
Client Characteristics • Product: What key capabilities are people looking for? Can we use our
existing products or do we need to develop a new one for this country?
- DigiBook’s Product: Uses an e-ink
electronic paper display that features 16 • Price: What price should the readers and books be sold at? What is
shades of grey. This allows for a 12 hour customers willingness to pay? Should we price the reader at a low price
long battery life and easy readability. so as to capture most of the market, and make margin on e-books?

- Locations: DigiBook has never sold a • Promotion: How should the e-reader be marketed? What promotion
product outside of the US. mechanisms should be used?
• Place/Distribution: Should the readers be sold through retail channels,
Competitive Dynamics internet or other alternative means?
- No competitors in the e-book or tablet • Selection of e-books: Is a wide selection of books available for this
space plan to enter this country country?
Prompts 1-3 – After the interviewee has laid our their structure, begin by asking the
question in Prompt 1. Once each prompt has been sufficiently covered, move to the
next prompt. After all prompts are complete, ask the interviewee to summarize their
findings. NOTE: Prompt 2 allows for creativity, so use your judgment when evaluating.

© 2011 Kellogg Consulting Club -- All Rights Reserved 137


Key elements to analyze

Prompt #1: Cust. Strategy Prompt #2: MKT Strategy Prompt #3: MKT Strategy
 Through research, we found several  DigiBooks is now considering how it should  Based on revised market estimates, we
segments. (Hand out Exhibit 1). We are only sell its e-book readers: through retail stores or decided to price the e-reader at $100 and
able to target one segment with our product. through the internet. target the Occasional Reader segment.
 Using a 3-year projection, which segment  How would you go about evaluating this  Using this information and Exhibit 2, can you
should the e-book reader target? decision? estimate the profit potential of each of these
sales channels? Which should we choose?
 What segment would you recommend to your
Notes on Exhibit 1 Notes on Prompt #2 Notes on Exhibit 2
client?

 If unclear, the interviewee should answer based on • Retail Channel Sample Responses:  If unclear, the interviewee should answer this with
Revenue Potential (i.e. Ignore probability of a 1-year Gross Profitability calculation for each of
 CONS:
purchase) assuming all reader sales happen the Sales Channels and back out the Upfront
immediately (Ignore TVM).  Lower margin due to value chain Investment. They should ignore all other costs
expansion such as SG&A.
 The missing data is that the average price of an e-
book is $10, for each of the segments, and that e-  Will take time and money to set up and  There is no missing data; however, the Market Size
books and e-reader have the same margin adds training costs of 20M people in the OR segment is needed from
 Using Exhibit 1, interviewee should calculate:  PROS: Exhibit 1 and the price per e-book of $10.
 SR Revenue = [(15 books x $10 X 3 Years) +  Should encourage Trial of the product  Using Exhibit 2, interviewee should calculate:
($200 x 1 e-reader)] x 8M People = $5.2B  Retailers can help with joint marketing  Retail Profit = [(10 Books x $10/book x 50%GM)
 OR Revenue = [(10 books x $10 x 3 Years) + campaigns + ($100/e-reader x 30%GM)] x (10M People x
($125 x 1 e-reader)] x 20M People = $8.5B  Retailers can help with customer service, 40% Penetration) = $320M - $20M II = $300M
 RR Revenue = [(5 books x $10 x 3 Years) + ($175 returns  Internet Profit = [(10 Books x $10/book x
x 1 e-reader)] x 10M People = $3.25B • Internet channel Sample Reponses: 50%GM) + ($100/e-reader x 60%GM)] x (10M
People x 10% Penetration) = $110M - $10M II =
 ANSWER: Segment to be targeted = occasional  PROS: Likely cheaper to establish, will result in $100M
readers (OR) with a Revenue Potential of $8.5B higher margins  ANSWER: Sales Channel to Use = Retail with a
 CONS: Hard to encourage trial profit of $300M

© 2011 Kellogg Consulting Club -- All Rights Reserved 138


Case 23: Health Coaches
By: David Wellner (Kellogg Class of ‘11), Edited By: Craig DePriester (Kellogg Class of ‘12)

Case Question

 Our client is a large national health care payer (health insurance company, think Aetna) exploring the launch of a new
disease management (“DM”) program to better serve its 5 million members. The idea is to hire and train a team of 8
“Health Coaches” to specialize in a single disease area (e.g., heart disease, diabetes, etc). Each Coach will manage a Quants.
portfolio of patients to reduce the costs of overall health expenditures (e.g., reminders to take drugs, provide limited
medical advice, suggested diet, etc). Studies show that once a month contact with each patient reduces health 6
spending by 5%, on average.
Structure
 Should our client launch the program? If so, what steps should it take?

Case tracker Fit Questions Guide to interviewer


Cust. Stgy
 Industry: B/E
Spend first 15 min on fit  The case tests the interviewee’s ability to probe and
Healthcare develop a customer segmentation, digest a relatively
 How do you feel about
 Level of Difficulty: complex chart, isolate the most critical information
working in a feedback-
Medium and determine profitability
intensive environment?
 Case Format:  The data provided by both exhibits should be
 Why is consulting a
Developing a new requested; try not to show the exhibits until need for
better career move for
product/Service the data is demonstrated
you compared to your
 Concepts Tested: next best option?  Strong interviewees should use common sense to
- Customer strategy make reasonable assumptions before you provide
- Break-even analysis required inputs

© 2011 Kellogg Consulting Club -- All Rights Reserved 159


Clarifying answers and handout guide

Clarifying answers to provide Guide to handouts


Competitive dynamics (not core to case) Before showing exhibits, interviewee should convey the essence of the case:
Are the costs associated with the DM program justified by the savings?
 With spiraling health care costs, the
industry is under pressure to innovate
Sample set-up:
new products that will control spending
Program Savings Program Costs Risks
 Assume client is first to market
- Customer segmentation - Salary and other - Do assumptions hold?
 Past attempts to purely automate DM by disease area and - Portfolio size/capacity - Competitive response
have yielded minimal savings cost per member (members/coach) - Regulatory, liabilities

Health Coaches Exhibit 1 – Hand out when interviewee establishes need for understanding client’s
 All activity conducted remotely via membership segmentation and/or exposure to disease areas. If he/she is not
phone/email headed there alone, you can ask “how would you segment the client’s members?”
 “What can we get out of this chart? Please let me know if you have questions”
 Typical profile is registered nurse that  Definitions (if needed): Group are employee sponsored plans (e.g. ,if you work
wants to work from home for McKinsey, you are in a group plan), Individual are non-groups (e.g., private
 It’s difficult to actually reach patients, contractors, unemployed, etc). 65+ (see asterisk below chart).
so Coaches can contact 8 members per  “Which segment is likely to generate the greatest per member costs.? Why?”
day (assume 25 days per month)  “Which disease area should we look at first?”
 Annual costs per Coach: $60K salary Exhibit 2 – Hand out when interviewee asks for medical cost data. Try to avoid
+20% other (training, benefits, laptop, handing out exhibit 2 until exhibit 1 has been discussed (hint: if interviewee leads
etc) with profitability, steer him/her to first think about the customer segmentation)
- There are no other program costs  “What can we do with this information?”

© 2011 Kellogg Consulting Club -- All Rights Reserved 160


Key elements to analyze

Segmentation and disease focus Program profitability


Using Exhibit 1, discuss which segments and disease areas are Leveraging all data (Both Exhibits), interviewee should determine if
most important to explore Health Coaches are profitable in each of the three segments

Notes to interviewer Notes to interviewer


Interviewee should choose to focus on the 65+ segment Cost per Coach: $60,000 + 20% = $72,000
 65+ (Medicare) patients are the sickest, followed by Base Salary + other = per coach per year
Individual Size of Portfolio: 8 25 𝟐𝟎𝟎
 Group members are the healthiest (younger, working) 𝑐𝑜𝑛𝑡𝑎𝑐𝑡𝑠 + 𝑑𝑎𝑦𝑠
=
𝒎𝒂𝒙. 𝒑𝒂𝒕𝒊𝒆𝒏𝒕 𝒑𝒐𝒓𝒕𝒇𝒐𝒍𝒊𝒐
𝑝𝑒𝑟 𝑑𝑎𝑦 𝑝𝑒𝑟 𝑚𝑜𝑛𝑡𝑕
 Sicker patients are likely to drive higher costs, which will
make them riper candidates for the DM program (i.e., the Savings for one portfolio of 65+ diabetics:
5% cost reduction will have a bigger impact) $300 4 5% 12 200 $𝟏𝟒𝟒, 𝟎𝟎𝟎
𝑎𝑣𝑔. 𝑥 𝑑𝑖𝑎𝑏𝑒𝑡𝑖𝑐 𝑥 𝑎𝑣𝑔. = 60 𝑥 𝑚𝑜𝑠. 𝑥 𝑝𝑎𝑡𝑖𝑒𝑛𝑡 = 𝒔𝒂𝒗𝒊𝒏𝒈𝒔 𝒑𝒆𝒓
Interviewee should choose to focus on diabetics (Assume all
𝑃𝑀𝑃𝑀 𝑓𝑎𝑐𝑡𝑜𝑟 𝑠𝑎𝑣𝑖𝑛𝑔𝑠 𝑝𝑜𝑟𝑡𝑓𝑜𝑙𝑖𝑜 𝑪𝒐𝒂𝒄𝒉 𝒑𝒆𝒓 𝒚𝒓.
are Type 2 for the purposes of this case)
 Diabetics make up the largest portion of sick members
Overall savings: $144𝐾 $72𝐾 $72𝐾 2,000 $𝟏𝟒𝟒 𝑴𝑴
 As a chronic disease primarily brought on by behavior, T2 − = 𝑥 =
𝑠𝑎𝑣𝑖𝑛𝑔𝑠 𝑐𝑜𝑠𝑡𝑠 𝑝𝑟𝑜𝑓𝑖𝑡 𝐻𝑒𝑎𝑙𝑡𝑕 𝒑𝒓𝒐𝒇𝒊𝒕
diabetics are most likely to benefit from DM program 𝑝𝑒𝑟 𝐶𝑜𝑎𝑐𝑕 𝐶𝑜𝑎𝑐𝑕𝑒𝑠 𝒑𝒆𝒓 𝒚𝒆𝒂𝒓
Number of 65+ diabetics: Conclusion:
20%
𝑥
40%
𝑥
5MM
=
𝟒𝟎𝟎, 𝟎𝟎𝟎  Profit is $72K per Health Coach, 2x cost of a Coach
𝑠𝑒𝑔𝑚𝑒𝑛𝑡 % % 𝑑𝑖𝑎𝑏𝑒𝑡𝑖𝑐 𝑚𝑒𝑚𝑏𝑒𝑟𝑠 𝟔𝟓 +  Based on PMPM diabetic cost data, Individual segment is
𝒅𝒊𝒂𝒃𝒆𝒕𝒊𝒄𝒔
break-even (50% less savings), Group segment is a loss

© 2011 Kellogg Consulting Club -- All Rights Reserved 161


Solution and recommendations

Solution & Recommendations


With 3-4 minutes remaining, give interviewee a moment to prepare a recommendation. Here is a strong sample:
 Client should launch the Health Coaching program, and first focus on diabetes for the 65+ Medicare segment
 The client should take the following steps:
- Launch a pilot program to prove out assumptions (e.g., 5% cost reduction, Coach portfolio capacity, etc)
- First expand to entire 65+ segment ($144M per year savings, a 2x return on each Health Coach)
- Consider introducing to Individual segment despite break-even (customer retention, moral rationale, etc)

Follow-up questions (if time permits)


 There are 650K Group diabetics left “uncoached.” Is there a way to make the segment profitable? Ideas include:
- More efficient DM program (e.g., Coaching at work, bi-monthly contact, automated correspondence, etc)
- Seek additional revenue sources (e.g., Employers might be willing to pay a fee, government support)
- Since 5% is the average savings, program can target members who will respond with savings well above 5%
 As first to the market, client plans to expand Health Coach program externally. Who should they target?
- It’s tempting to suggest that the client should market to 65+ Medicare patients with diabetes, since this is
where the program yields the greatest savings. While it’s good that we have lowered the cost of older
diabetics, client should keep in mind that more diabetic members will increase overall health care costs,
considerably – (still almost 4x after savings)
- An instinctive interviewee might suggest an alternative: Client should sell its Health Coach service to other
payers. But how would you price that? What are the challenges (e.g., regulatory, info sharing, etc)

© 2011 Kellogg Consulting Club -- All Rights Reserved 162


Exhibit #1: Client’s member segmentation by health condition

* Members 65 years of age and above. Known as “Medicare Advantage”, premiums funded by government

© 2011 Kellogg Consulting Club -- All Rights Reserved 163


CASE 3:
ORGAN DONATION CASE
Firm Style Interview Round
McKinsey 1/2
Case Question:
One of the many functions of the New York State Health Commission is to coordinate
organ donation amongst the state’s many hospitals. In recent years the demand for
organs has been greater than the supply. As a result, many patients die each year because
there are not enough organ donations. The Health Commission has hired McKinsey to help it
determine how to increase the number of annual organ donations in New York.
For the purposes of this case, assume that only New York residents are involved as donors
and recipients.
Question 1: What are the factors and drivers that determine the number of
organs donated in New York each year?

Clarifying Information
Provide the following answers only if the interviewee asks the corresponding questions.
Information

In New York, organs will be harvested from terminally ill or injured patients just before death only if
they are a registered organ donor or if the hospital receives permission from the next of kin. New
York residents may choose to register as organ donors when they apply for a driver’s license.
ORGAN DONATION CASE
Sample answer:
(Terminal patients*% Registered donors *organs/donor) + (Terminal patients*% Not
registered*% Family consent*organs/donor) = organs donated

“The number of organs donated is determined by the number of terminal patients


times the percent who are registered donors times the number of organs per
person, plus the number of terminal patients times the percent that are not
registered donors times the rate of family consent times the number of organs per
person.

“The number of terminal patients is driven by the age of the population, changes
in life spans, medical technology, the size of the population, the overall rate of
natural death, plus the rate of accidental death, such as car accidents. The
percentage of registered donors is driven by awareness of the donor program,
the ease of registering, whether it is opt-in or opt-out, awareness of the benefits of
organ donation and personal things such as religious beliefs. The rate of family
consent is also driven by awareness of the benefits of organ donation and things
such as religious beliefs, along with the information provided by doctors in a time
of great grief and assurance that organs will only be harvested after death is
certain. The number of organs donated per person is driven by the average
number of healthy, usable organs in a donor.

While we’ve identified a number of factors, it’s important to note that we can only
really influence the rate of registered donors and the rate of family consent.”
ORGAN DONATION CASE

Note: This question seeks to test a candidate’s breadth of thinking. A


good answer will be logical and structured. It will examine the factors
as an equation, though not necessarily exactly as presented here. It
will be thorough and should include 3-4 drivers for each factor. A great
answer examines the implications of the analysis – ie. There are only
two factors that we can influence.
ORGAN DONATION CASE

Question 2: The team has decided to focus on increasing the number of


registered donors and is specifically interested in kidneys donations. The
Health Commission knows that it needs 9,200 kidneys per year. What
percent of New Yorkers need to be registered donors in order for 9,200
kidneys to be donated in a year?
The team has already collected some useful information: The population of
New York is 10 million. The percentage of people that become terminally ill
or injured each year is .1% (one-tenth of one percent). The percentage of
families that give consent to harvest organs is 10%.

Sample answer: The candidate should setup and solve the following equation.
• RD = Percent of New Yorkers who are registered donors. (10,000,000*.001*RD)
+ (10,000,000*.001*.1*(1-RD)) = 4,600
• (10,000*RD) + (1,000*(1-RD)) = 4,600
• 10,000RD+1,000-1,000RD = 4,600
• 9,000RD=3,600
• RD=3,600/9,000
• RD=.4=40%

Note: Set the equation equal to 4,600 because there are two kidneys per
person.
4,600*2=9,200.
ORGAN DONATION CASE

Note: There are other possible ways to setup the equation. What’s
important is that the candidate takes a structured, analytical
approach to the problem and doesn’t panic. The candidate should
walk the interviewer through the math, either as she solves the
equation or afterwards. If the candidate gets stuck, give appropriate
hints.

A great answer will include implications of the data. For example, they
may conclude that 40% seems reasonable based on their personal
experiences knowing people who are registered donors. Anything to
show initiative and critical thinking without prompting is good.
ORGAN DONATION CASE

Question 3: What are all of the things that the Health Commission
should consider doing to increase the number of registered donors?

Sample answer: “First, we should conduct benchmarking studies. Internal


benchmarking will reveal what they are currently doing well to drive
registrations, and external benchmarking of other states or countries will
uncover new ways to increase registrations. Second, we should conduct a
segmentation of current and prospective registrants. We’ll want to
determine the segments where we are strong so we can continue to
cultivate them, and at the same time find new segments where few people
are registering. Third, we should conduct a series of marketing
communication tests to determine appropriate messaging and channels to
increase awareness and drive registrations. We should consider testing
media such as TV, radio, online, direct mail, and may also be able to
leverage the word of mouth of our current customers. We’ll also want to
develop messaging that increases the awareness of the donor program,
the benefits of donation and the safeguards that are in place. Finally, we
need to optimize our distribution channels. Currently, people can only
register when they get their driver’s license. We should add ways to
register, such as any time someone goes to the doctor or hospital. We
should also make it easier to register. For example, if New York currently
requires people to explicitly opt-in, we can change to an automatic opt-in
with an explicit opt-out.
ORGAN DONATION CASE

Note: This question is designed to evaluate the candidate’s depth of


thinking. The above plan is a sample only and there are other
approaches. A good plan will be MECE, with discrete categories (ie.
“Benchmarking”) and multiple items within each category. The plan
should demonstrate common sense and sound business judgment.
ORGAN DONATION CASE

Question 4: The team conducted a customer segmentation and


your Engagement Manager has asked you to draw some initial
conclusions from the data (give candidate the exhibit from the end
of the case – candidate should clarify that the data are in millions
of people). What would you tell her?
Follow up question: Which segment represents the best opportunity
to increase RDs?

Sample answer: “It looks like the Caucasian segment has the largest
number of potential new registrants, but at the same time is already highly
penetrated. The African-American segment is the opposite: low-
penetration but low upside. The Hispanic segment has a slightly higher
penetration than African-American, but is a bigger population. Based on
this data alone, I would target the Hispanic segment because of the low
current penetration and large population. Some of the risks include
overcoming the language barrier and dealing with cultural differences.”
ORGAN DONATION CASE

Note: The data table is designed to present the Caucasian, African-


American and Hispanic segments as strong possible targets for different
reasons. The candidate should reason through the pros/cons of
targeting each segment and come to a firm hypothesis based on this
data. It’s more important that the candidate selects a target for good
reasons and less important that she selects Hispanic.
ORGAN DONATION CASE

Question 5: The team has decided to focus on increasing the


registration rate among Hispanics. What are some things we can
do to increase registrations of Hispanics?
ORGAN DONATION CASE

Note: This question is designed to test a candidate’s creativity. Hence,


a sample answer is not provided. It is likely that the candidate will
suggest things such as community outreach, Spanish language
marketing materials/registration, etc., classic consumer marketing
tactics, having vans drive around Hispanic communities encouraging
people to register, etc. A good answer will have a long list of ideas that
include typical business tactics along with creative, interesting
suggestions. The interview should push the candidate by repeatedly
asking “What else?”, until the candidate can no longer think of things,
and to determine how they respond to pressure
ORGAN DONATION CASE

Recommendation
Question 6: The head of the Health Commission just walked in and
wants a recommendation for what he should do to quickly reach
Hispanics. What’s your recommendation? Start talking now.

Sample answer: “Clearly, there is a big opportunity with


Hispanics. We’ve identified a number of things we can do to
increase registrations, and since we’re focused on the short
term, I recommend that we do X, Y, and Z, because of A and
B.”
ORGAN DONATION CASE

Note: A good answer should be short (20 seconds), organized and


forceful. The candidate should be firm and not hedge her answer. The
recommended steps will likely be things discussed in Question 4, and a
good answer will choose tactics that, logically, will make the biggest
impact.
ORGAN DONATION CASE

Exhibit 1

Registered Population(M)
Donors(M)

Caucasian 2.600 5.500

African-American 0.075 1.500

Hispanic 0.250 2.500

Other 0.100 0.500


CASE 4:
CENTRAL POWER
Firm Style Interview Round
McKinsey, BCG, AT Kearney 2

Case Question:
Our client is a regional electricity monopoly. Due to regulation, its market will be opened to
competition next year on January 1. There are no actual competitors at this time; however,
the client would like ideas on actions to take in the short-term in order to better prepare for
potential competitors. They have no control over pricing and cost-cutting during this time
period.

Clarifying Questions & Answers


Provide the following answers only if the interviewee asks the corresponding questions.
Question Answer

Do we know of any potential competitors? None at this time. Central Power is facing the
threat of market entry.
Do you have any information on pricing? Not really; pricing is controlled by a
government, so it is beyond our control and will
be the same for any competition.
Cont’d on next slide
CENTRAL POWER

Clarifying Questions & Answers (cont’d)


Provide the following answers only if the interviewee asks the corresponding questions.
Question Answer

Are there other markets Central Power can None at this time. Our engagement is focused
enter? on their home market.
How is Central Power organized? Central Power has three divisions:
1. Sales and General Administration
2. Generation and Transmission
3. Maintenance and Repair
How many employees are in sales? About one for every 1000 customers.

Does sales, customer service treat certain No, they treat everyone equally.
customer segments differently?
If there is a power outage, does Maintenance No, they dispatch in the order of the call they
and Repair fix industrial clients first? receive.
Has Central Power ever advertised or had a Not in recent memory.
marketing campaign?
Is Central Power undertaking any capital Not in the next year.
investment?
CENTRAL POWER

Framework / Structure
This case is about customer retention within the home market. Since the client is a
monopoly, market share is 100% and customer acquisition is irrelevant.

The analysis may include, but is not limited to, the following areas:
• Barriers to Entry
• There is no actual competition to specifically target
• The government is deregulating, so there are no possible external barriers
• Internal barriers are possible, such as increasing switching costs and
commercial attractiveness to customers
• Anticipation of Competitive Attack - Once the candidate sees the exhibit and
realizes how attractive the industrial clients are, they can anticipate how a
rational competitor would enter the market.
• Market Information
• Revenue
• Customer Segmentation
• Stakeholder Reaction - How will employees, unions, or local government
leaders react to our changes? (There is no set answer to this, just ask the
candidate what they think and see if it is logical and structured.)
CENTRAL POWER

Note: Strong plans will anticipate the need to be competitive and draw on
analogous companies or situations.

Less experienced candidates gravitate towards issues of pricing and cost-cutting.


This is not a profitability case. Weak candidates will often explore growth outside
the home market, such as expansion into adjacent markets. While interesting, this
does not address the primary problem of what to do in the home market before
deregulation. Avoid these tangents.

Steer the candidate to customer retention.


CENTRAL POWER

Phase 1
Qualitative reaction and quantitative analysis of revenue data
Purpose:
1. Test rapid understanding of a basic chart
• Most candidates ask about revenue very early. Show them the attached
exhibit and immediately ask, “What is your reaction to this data?” Strong
responses notice the differences between the customer segments.
• The “historical data” is irrelevant. If a candidate starts to ask about it or discuss
trends, inform them that is a factor or population and government-
determined prices, which are both beyond the control of the company.

2. Test basic arithmetic


Test basic math by asking the candidate (if they haven’t already done so) to
calculate the share of total revenue per segment and each segment’s revenue
per customer. If a candidate didn’t notice the differences between customer
segments already, this is a second chance. Note: I don’t have a problem if a
candidate wants to round the total revenue to $250 MM from $258 MM to
simplify the math…
CENTRAL POWER
Industrial Commercial Residential
# of Customers 150 10,000 100,000

Revenue $150 MM $60 MM $48 MM


Revenue per Customer $1,000,000 $6,000 $480

Residential Customer Equivalents 2083 13 1


Fraction of Total Revenue 150/258 = 58% 60/258 = 23% 48/258 = 19%

NOTE: the math is really basic, but there are a lot of zeros that tend to trip up
candidates. Most strong candidates write out the problem to avoid error, even if
their math is facile. Now is a good time to diagnose how well the handle orders
of magnitude. If this is a problem, here are some suggestions for the candidate.
CENTRAL POWER

3. See if candidate grasps key insight regarding higher value of industrial


customers and uses it for convergent logic in Phase 2.

• By now, candidates should realize that losing a single industrial customer


is similar to losing more than 2000 residential customers. Churn of industrial
and commercial customers is lower too and it is easier to approach a few
hundred “B2B” customers than one-hundred thousand individual
homeowners, apartment renters, etc.
• If a candidate still needs help, ask them, “Let’s turn the situation around.
Based on this data, if you were a competitor, which segment would you
target?” They should realize industrial customers. If the candidate doesn’t
verbalize that Central Power should target or defend the industrial
customers, then ask, “OK, based on your insight about a potential
competitor, what should Central Power do to pre-empt them?”
CENTRAL POWER

Phase 2
Generation of ideas regarding customer retention improvements.
• Now that the candidate grasps that any action should be biased towards the
smaller and higher value segments, it is time to generate actual ideas. This part of
the case is extremely unstructured- there are no more data sheets or obvious
answers.
• Assume Central Power is decades behind the rest of the business world- they have
no website, send out massive paper bills, treat customers with the finesse of a huge
monopoly, etc.

Purpose
Test candidate’s ability to generate their own ideas with structure and logic
• The prime differentiator between weak and strong candidates is their ability to
have an exploratory conversation that exchanges ideas.
• Strong candidates will announce that they want to discover ways of improving
customer attachment (or retention, stickiness, etc.) to Central Power and begin to
explore the structure of the company looking for areas to improve. They use a
conversational style to ask questions. They typically draw on personal experience
as residential customers of electricity (or another utility).
CENTRAL POWER

• Also, they may ask about the organization of the company or use a
“value chain concept” to structure their questions regarding the
organization of the company. At each part of the company, they ask
questions about behavior, customer service, etc. to see if we focus
on industrial clients. Great candidates really have some out-of-the
box ideas, such as energy savings, simplified billing, special websites,
improved terms for accounts receivable, etc.
• You will know you have a strong candidate when they move
methodically through the company and use their imagination. Let
them continue on as long as possible by asking, “What else?” This is
an opportunity to test a candidate’s ability to structure their
reasoning and demonstrate conversation skills.
• For instance, they should be able to tactfully move the investigation
to a new part of the company when they have exhausted their
imagination. These meta-skills are more important than specific
ideas, so feel free to make up whatever you want about Central
Power’s operations.
CENTRAL POWER
Here are some ideas to facilitate customer retention by building a stronger relationship or
making switching more difficult:
a) Sales and General Administration
i. Sign customer contracts
ii. Increasing credit terms / days allowable for accounts receivable
iii. Increasing the size of the sales force
iv. Allocating sales staff to specific segments, since the value proposition is different
v. Allocate special personal to high-value accounts
vi. Reduce or simplify the billing process
vii. Create a website for customers to manage accounts online.
b) Generation and Transmission
i. Consider building substations or redundancy that improves reliability or quality for
industrial clients.
ii. Work with industrial clients to improve their energy efficiency
c) Maintenance and Repair
i. Fix outages by order of customer priority, not first-come, first serve. (Besides, Industrial
clients are more sensitive to power outages than residential customers)
ii. Preposition repair equipment near clusters of industrial clients to reduce repair time.
iii. Proactively visit clients to inspect their infrastructure or provide engineering advice
CENTRAL POWER

Weaker candidates face typically fall in two groups:

Group 1 – Lost in the Sauce: Some candidates have no idea how to


systematically generate ideas. Try and prompt them by asking about their
own experience with a power company, or ask them to think of the value
chain.
Group 2 – Lone Rangers: Other candidates will have a systematic approach,
but will make assumptions and monopolize the conversation. They declare,
instead of explore, their way through this phase. They will make up their own
data and draw conclusions. For example, they might say, “Our client must be
bad at customer service, so I would consider dedicating people to high-value
clients.”

A real conversation should flow like this: “Do we know how customers feel
about service?” Response: “They hate it. The number one complaint is
“every time I call, I speak to someone new.” Candidate: “Perhaps we should
dedicate some sales staff to high-value clients that we want to retain the
most.” Response: “Good idea, what else?”
CENTRAL POWER

There are two other meta-skills to evaluate:

• First, does the candidate explain their structure and line of


reasoning? It should be “transparent thinking”. I use an analogy of a
tour-guide. A strong candidate tells me where we are going with
their plan, takes me through it, keeps me from getting lost when we
go outside the plan, and brings me back.

• Second, due to all the ideas being generated, a candidate needs


good note-taking skills. Strong candidates will circle ideas on their
plan or make a list in the corner of the page. When it is time to make
a recommendation, they can quickly cover ideas at a glance. Weak
candidates will attempt to recall ideas off the top of their head,
usually without total success.

End the case when you are out of time or the candidate is out of
steam.
CENTRAL POWER
Recommendation
The recommendation should include the following:
• The answer – We have developed several ideas for improving customer retention; all
priorities should be biased towards Industrial customers.
• Next steps – Analyzing costs, potential ROI, and effects on stakeholders for particular
customer retention initiatives.

Strong Recommendation
“By investigating the operations of your company, we have developed several ideas for
strengthening customer retention in the face of competition. No matter which actions are
undertaken, there should be a bias towards industrial customers who are more valuable as
a group and on a per-customer basis. If any of these actions seem attractive to
management, we can undertake more detailed analysis to support

Tepid Recommendation
Many times, students will say the same thing as above but with too much detail, too much
length, or without enough structure. When they are done rambling, I ask them to say it
again in 30-seconds or in three sentences. Sometimes it takes a few attempts, but it is more
worthwhile then telling them later during feedback that their recommendation wasn’t good
enough. (Close only counts in horseshoes and hand-grenades.

Weak Recommendation
Weak recommendations are missing a unifying aspect, such as customer retention. They
also fail to include the key insight regarding the preferential value of industrial customers.
Further, they may include something totally off-base, like cost-cutting or pricing.
CENTRAL POWER

Questions to Further Challenge the Interviewee

1. How do you think the employees (or another stakeholder) will feel
about these changes?
2. What do you think it will be like working for a client that lacks a
history of customer service?
CENTRAL POWER

Exhibits
CASE 9:
BIG GREEN BIOFUELS
Firm Style Interview Round
McKinsey 2

Case Question:
Big Green Fuel Systems, a large provider of fuel additives required for the
production of gasoline, has recently developed a substantially improved form of
ethanol that adds 20% efficiency (measured as miles driven on a single gallon of
ethanol). This version of fuel is costly to produce but the market is large and
growing more rapidly as increased amounts of ethanol are being used in fuel
blends. The company has engaged your team to help determine the viability of
this new product, and, if it is viable, how to go about the process of launching it.
Clarifying Questions & Answers
Provide the following answers only if the interviewee asks the corresponding questions.
Question Answer

Is the company working in the US only? Yes, please concentrate on the US market only.

Does the current infrastructure exist to make It’s compatible with other infrastructure in the
and support this product? external value chain, but Big Green does not
yet have the capacity to mass produce it.
Cont’d on next slide
BIG GREEN BIOFUELS

Clarifying Questions & Answers


Provide the following answers only if the interviewee asks the corresponding questions.
Question Answer

What criteria does Big Green want us to use to Please develop your own criteria.
determine the viability of the project?
What is the value chain for this product from Big Ethanol is delivered from manufacturers to fuel
Green to the consumer? blenders, who then sell and deliver in bulk to
filling stations, who in turn sell it to consumers.
Does Big Green have a patent on this Yes, they have a patent.
technology?
BIG GREEN BIOFUELS

Structure / Framework
This is a value chain analysis case. The analysis may include, but is not
limited to, the following areas:
• Standard valuation (ROI + intangible benefits). What is the initial and
recurring investment?
• What is the incremental profit from this venture? What can be gained
beyond the sale of ethanol?
• What barriers exist for this product? How will the value chain
respond? Can we make it?
• What is the standalone value of this product, and what is its value to
other producers? Can we sell it?
• What will existing competitors do in response?
BIG GREEN BIOFUELS

Strong Plan
Includes valuation of the product as part of Big Green and at least
considers its value for other firms. It is necessary to consider barriers to a
successful launch (both internal capability and external pressures). Also
a strong plan considers the competitive response and how it is
manifested throughout the value chain. Intangible benefits (spin-offs,
positive PR, etc.) should be considered as well.

Weak Plan
Valuation is important but not strong enough on its own. Failing to
consider costs and investments (both fixed and working capital) is a
shortcoming. Failure to consider the possibility of selling the product to
another, more capable company shows lack of creativity. Intangible
benefits (hard to quantify) are a strong opportunity to showcase
creativity as well. Failure to consider competitive response and the full
impact on the value chain would also result in a negative hit.
BIG GREEN BIOFUELS

Calculations
Interviewer: Thank you for sharing your approach. Now I would like you to size
the market for me. But I will give you some data. First, there are two ethanol-
gasoline blends used in the US, E-85 and E-15. E-85 is 85% ethanol at 10% of the
market and E-15 is 15% ethanol at 90% of the market. The number of miles driven
in the US last year was 1,800 billion miles. Please give me the market in gallons for
ethanol.

This is fairly simple, but it’s important to first develop the approach (gallons of
ethanol = (1,800 billion total miles / X average gas mileage) * 85% ethanol
content * 10% of the market + (1,800 billion total miles / X average mileage) *
15% ethanol content * 90% of the market.

A key is seeing that the X mileage is missing and is needed, so the interviewer
should ask for it. This will yield a response from the interviewer of 20 miles per
gallon on average. Thus: (1,800 billion / 20) * .85 * .10 + (1,800 billion / 20) * .15 *
.90 = 19.8 billion gallons of ethanol.

Expected Insight: One should notice that there’s not enough information and
know to ask for the average mileage in the US (20 miles/gallon). Also, one should
ask about the 20% additional efficiency, but the interviewer will instruct the
interviewee to ignore it. Once the interviewee calculates the answer, he or she
should remark about the size of the market, and immediately want to know how
fragmented it is, and will be told it’s highly fragmented.
BIG GREEN BIOFUELS

Interviewer: Good. Now, the analysis has determined that there is no savings by
using this new fuel blend. Basically, the fuel costs exactly as much to develop
and deliver as can be gained by selling it. But the CEO has staked his reputation
on this product, so he is committed to delivering it. Please explain how you
would best appeal to each level in the value chain in order to best promote this
improved product.

Expected Insight: The value chain should be looked at from source to


consumer:
• Selling to blenders: the fuel blenders will have to blend 20% less (but will pay
for it as a price premium). However, they could use it as a competitive
advantage by differentiating on its basis. Additionally, it could reduce its
distribution costs (delivering less to filling stations). Also, blenders would have a
competitive advantage by being positioned for potential future government
regulation.
• Filling stations: differentiating factor for eco-sensitive consumers. Point of
positive public relations. Appeals to drivers who want to spend less time filling
up. Less refills needed from the blender, so less delivery costs. Could also make
it possible to distribute gas from smaller tanks, making it possible to put new
stations in formerly prohibitive locations.
• Consumers: eco-friendliness. Higher cost of fuel set off by less need to fuel up
(efficiency) and there is an additional value add since they’ll spend less time
at the gas station.
BIG GREEN BIOFUELS
Recommendation
The recommendation should include the following:
• The answer – Since the interviewer will make it clear that the CEO is intending to roll out the
new fuel, this is a given. The main points from the second question, regarding the value
chain, should be included as supporting evidence.
• The number(s) – The ethanol market number is optional and may not do anything to add
value to the argument.
• Risks or considerations – Rolling out the new fuel could cause a price-perception problem
(higher cost to fill up). Attempts to fix this through marketing efforts could also drive
marketing costs. There is also the consideration of what it means to release a new fuel,
including risk that the fuel is more corrosive than other existing fuels.
• Next steps – Next step would be launching a pilot program, including marketing efforts, to
test the fuel for its efficacy. Lessons learned would be applied to a gradual national roll-out.

Strong Recommendation
The CEO has indicated that he wants to roll out the new fuel. The best way to do this is to
appeal to blenders and gas stations on the basis of reduced delivery costs and enhanced
public relations, and appeal to consumers on the basis of eco-friendliness and fewer required
fill-ups. The biggest risk is alienating consumers due to the higher cost perception and
alternatively the marketing cost of mitigating this perception.

Weak Recommendation
The market size is 20.9 Billion gallons of ethanol each year. Thus Big Green should roll out the
new product because it’s more efficient and could cut associated costs and make green
consumers happy.
CASE 14:
ELECTRIC CAR MANUFACTURER
GROWTH
Firm Style Interview Round
McKinsey 1/2

Case Question:
Our client is a start-up electric vehicle manufacturer that has built a
prototype all-electric vehicle and is interested in mass-producing it for the
U.S. market. We have been hired to help them think about this opportunity
Clarifying Questions & Answers
Provide the following answers only if the interviewee asks the corresponding questions.
Question Answer

What type of vehicle(s) has the client It is a light-duty all-electric truck. They have
developed? manufactured 3 identical prototype vehicles to date.
What are the client’s existing Minimal. The current manufacturing facility is little
manufacturing capabilities? more than a large R&D lab.
Where does the client hope to sell their In the U.S. market. Nationwide.
vehicles?
Cont’d on next slide
ELECTRIC CAR
MANUFACTURER GROWTH
Clarifying Questions & Answers
Provide the following answers only if the interviewee asks the corresponding questions.
Question Answer

4. How many vehicles does the client want to 150,000 vehicles in Year 1.
produce in Year 1 of manufacturing?
5. How much will the vehicle retail for? Pricing has not been determined yet, but will
likely be in the $35k-$50k range.

Question 1
How would you think about the potential market for this vehicle in the U.S. (i.e. market
segments)?

Strong Answer
Breakdown of different customer segments in the U.S. auto industry (i.e. luxury, hybrid,
vehicle class (SUV, sedan), etc).
Candidate should recognize that the potential market for the vehicle will depend on the
price, its physical characteristics (i.e. amount of cargo/passenger space), range of the
vehicle, choice and availability of distribution.
ELECTRIC CAR
MANUFACTURER GROWTH
Weak Answer
A weak answer will contain only one or two examples of different dimensions that
consumers use to purchase vehicles. Candidate should expect to get pushed for “what
else” if your answer is too thin.

Question 2
How many employees will be required to build the desired first year production run
of 150,000 vehicles? The following information is known about the production
process:
• The vehicle requires approximately 600 unique assembly steps.
• Each assembly step takes approximately 30 seconds.

Strong Plan
Student needs to ask some clarifying questions:
• How many hours per day (week) would the average employee work?
• 10 (50)
• How many weeks per year does an average employee work?
• 50
ELECTRIC CAR
MANUFACTURER GROWTH
Weak Answer
Candidate makes assumptions without clarifying with the interviewer.

Calculations
Number of employees needed = 150,000 vehicles x 5 hours per vehicle/(2500 hours
per employee per year) = 300 employee

Question 3
What options should the client consider for scaling up manufacturing to mass-
produce the vehicle? What are the pros and cons of each option?

This question seeks to test a candidate’s depth of thinking. A good answer will be
logical, structured and will discuss drivers.

Strong Answer
A strong answer will discuss the pros and cons of various manufacturing options. For
example they could build a manufacturing facility in the U.S., build a manufacturing
facility in a low-cost SE Asian country, or outsource production (under contract/joint
venture) in the U.S. or overseas. Items to be addressed include labor costs, shipping,
quality (actual or perceived), regulation, public reaction, proximity to suppliers, etc.
ELECTRIC CAR
MANUFACTURER GROWTH
Weak Answer
A weak answer will cover only one or two areas addressed above. Having at least three pros
and three cons for each option covered is advisable.

Question 4
Assume that the client has decided to build a manufacturing facility in the United States,
what are some strategies that they could use to reduce costs. Discuss the trade-offs of each
approach. Note: This question is designed to evaluate the candidate’s breath of thinking.

Strong Answer
There are many different ways to address this question. The “most-right” answer is to consider
the trade-off between more automated technology and more manual labor. The
automated technology will require a substantial initial investment and high fixed costs but will
reduce the direct labor (variable) costs. Less automation will mean lower upfront and fixed
costs but higher variable costs. Other potential answers include choice of location (right to
work state vs. union), engineering optimization (reduce the number of steps vs. vehicle
customization), etc.

Weak Answer
A weak answer will not address multiple cost-savings approaches and the trade-off involved
with each one.
ELECTRIC CAR
MANUFACTURER GROWTH
Question 5
What would you advise the CEO to do regarding the decision to mass-produce an
electric vehicle next year?

Strong Answer
There is no single right answer. Some additional insights that have not already been
covered by previous questions include: advising the client to consider licensing/selling
the technology to a major auto-industry player with established manufacturing and
distribution, forming a JV or strategic alliance with an established player, launching a
pilot program to work out kinks in the technology and gauge market reaction. A
good recommendation will have a decisive approach with supporting argument and
address the risks and ways to mitigate risks of the recommended approach as well as
next steps.

Weak Answer
A weak recommendation brings nothing new to the conversation. Simply recapping
some of the items uncovered in previous questions is not a strong recommendation
CASE 16:
BIG TRUCK COMPANY
Firm Style Interview Round
McKinsey 2

Case Question:
The client, Big Truck Co., is a large, US based manufacturer of heavy duty trucks, also known
as semi-trucks. They make two types of vehicles, semi-trucks designed for pulling trailers and
trucks or chassis’ used for cement trucks, waste haulers, etc. The company has three
marquees (brands), all of which manufacturer vehicles for both types of vehicles. The three
brands are the result of acquisitions done in the past five years. The company is profitable,
but profit margins have been declining slightly over the last few years. Additionally, the
market is forecasting a significant market decline in the next few years, which brings great
concern to the management of the company.
Clarifying Questions & Answers
Provide the following answers only if the interviewee asks the corresponding questions.
Question Answer

How large is the company in terms of revenue? $30 billion

Can we clarify that the objective is to preserve Yes


profitability as the market declines?
Does the company sell into international Yes
markets?
BIG TRUCK COMPANY

Framework / Structure
Question 1: The CEO has hired McKinsey to help the company better
understand their position and to look for solutions for the coming years.
How would you structure the problem for the CEO?
• Sample “Poor” Answer: I would want to examine the revenue, starting with volume and then
looking at price. Second, I would want to look at costs, better understanding the variable and
fixed costs.

• Sample “Good” Answer: This is clearly a profitability question. In order to understand this
problem, we need to examine the cost structure of the business and determine where we can
be flexible in an ensuing economic downturn. Additionally, we may want to understand ways
of adding to our top line. Starting with the revenue, I would want to better understand the
product mix. Perhaps there is opportunity to upsell for better profitability. Second, I would want
to understand our sales model (dealership, wholesale, fleet, etc.). On the cost side of the
equation, I would want to examine any additional synergies we could achieve from our recent
acquisitions. We could also look at our fixed costs and overhead and determine if new
investments may be worth it to save costs in the long run. Finally, I would want to look at the
variable costs and see if we can reduce our labor or input costs on a per truck basis.

• Sample “Great” Answer: [A great answer would include all the elements of a good answer, plus
the following additional ideas.] From a revenue standpoint, analysis of our three marquees
would be useful. Perhaps we are cannibalizing our own brands and need to consider a drastic
reorganization. Additional, analyzing the competitive marketplace could provide significant
information on how we can achieve better profitability. Within costs, operational efficiencies
and lean processes could be a way to achieve costs savings. One thing to watch out for,
however, is regulatory changes, most likely emissions rules, that could dramatically change our
costs. We should be cognizant of this and other external factors in everything we consider.
BIG TRUCK COMPANY

Question 1 (cont’d):
Color Commentary: While a profitability framework does work well in this
case, the specific bullets beneath revenue and costs should truly address
this company in particular and the issues it will face. Here is a potential
framework that will serve the interviewee well in this case:
BIG TRUCK COMPANY

Calculations
Question 2: The team decided to take a closer look at the costs of
producing a particular line of trucks. Take a look at Exhibit 1. What is
the potential savings for Big Truck Co. were we to be best in industry in
regard to our cost structure?
BIG TRUCK COMPANY
Question 2 (cont’d):
The interviewee should be told to leave the Manufacturing Mix constant in
the calculations.
• Sample “Good” Answer: If I understand this correctly, we are basically
indexing the costs of both manufactured goods and outsourced parts against
that of our competitors. Given that, it looks like we could save 20% in our in-
house manufacturing ((100-80)/100) if we had the same cost levels as
competitor B. On the outsourced parts, it looks like we could save ~33% ((120-
80)/120 if we matched the pricing competitor A is achieving. Given we know
our product mix, we know how much of the $40,000 is being spent on both
areas. I can then calculate the new cost at the “best in the business” cost
structure. [Performs calculations]. It appears that we can save approximately
$9,600/truck or 24% of our original cost on a blended basis.

• Sample “Great” Answer: [Same initial answer as above.] While this savings is
great, I want to better understand the savings to the company. Do we know
how many units are manufacturing in this product line a year [answer of 5000
units given]. Given that, it appears we could save $48 million dollars a year.
For a $30 billion company, that is about 15 basis points of operating margin
before accounting for taxes.
BIG TRUCK COMPANY

Question 2 (cont’d):

Color Commentary: The candidate will likely struggle with understanding


how to calculate the savings as the indexing is an odd way of thinking
about the market. Be prepared to nudge the interviewee to keep the
product mix constant. Additionally, many interviewees may use 80%/100%
for the outsourced parts, instead of 80%/120%. Watch for this and guide the
interviewee toward a better answer. Obviously, the great answer brings in
the context to the company as a whole and truly seeks to understand the
impact of the savings.
BIG TRUCK COMPANY

Question 3: The team decided to take a closer look at the costs of


producing a particular line of trucks. Take a look at Exhibit 1. What is the
potential savings for Big Truck Co. were we to be best in industry in regard
to our cost structure?
• Sample “Great” Answer: If we structure this out, I think we can come up with multiple
ideas as to how we can remove costs for this system. Let’s look at it this way:
In-House Manufacturing
• Lean/Six Sigma evaluation, continuous improvement
• Labor evaluation – are we union? What can we change?
• Commodities/inputs – LT contracts?
• Facilities – are they old/new? Can we upgrade? Are we at capacity? Are the
flexible?
Supplied Parts (Outsourced)
• Can we improve our contracts?
• Is our delivery/distribution poor and costing us additional dollars.
• Are we scaling properly across our marquees? Can we leverage the
volume/standardize parts?
Other
• Can we improve by changing the mix of outsource versus manufactured parts?
• Can we vertically integrate? Buy suppliers?

• Sample “Good” Answer: [The interviewee will list a number of the items listed in the great
answer, but will lack completeness and structure in comparison.]
BIG TRUCK COMPANY

Question 3 (cont’d):
Color Commentary: A great answer will really go through this problem in a
structured way. Additionally, each answer should be listed as a
conversation with the interviewer. Ask outright about some of the ideas.
Perhaps they have been tried already. The interviewer can engage as
much as desired in this question guiding the interviewee to specific ideas if
desired.
BIG TRUCK COMPANY

Question 4: The team also identified another area that it thinks there is
potential for savings. The company has a significant used truck business
and has contractual obligations to purchase trucks back form customers
who bought them new. Take a look at Exhibit 2. What analysis can you do
from this data? What is the potential for Big Truck Co.?
Outline of Calculations:
BIG TRUCK COMPANY

Question 4 (cont’d):
• Sample “Good” Answer: After analyzing this program, it is clear that we are mispricing
the trucks intended to be bought back. We are offering to buy trucks at $25,000, but at
no point in the next three years are the trucks worth that much. While this may be a
good program and offer us other profits as part of a used truck business, we are taking a
large loss with this program. Based on the calculations, we are projected to lose $125
million dollars and I suspect we are selling trucks under these same contracts every day,
only further exacerbating the problem.

• Sample “Great” Answer: [After answering the good answer, additional points like this
could be made]. Although it seems obvious that we should seek to change these
practices, I would want to better understand why we are in the position that we are in. Is
there an advantage to selling at a loss because it translates into larger profits in our used
truck business? Does this create goodwill with our customers and draw them to buy more
new vehicles at a greater margin? Understanding these issues more fully would better
help me understand how to create a positive solution in regard to this issue.

Color Commentary: This problem has fairly straightforward math and


shouldn’t take the interviewee long to see what the potential loss would be.
The great answer should not only bring to light the scale of the potential
loss, but be inquisitive as to why this might be the case. A good interviewee
should also guide the interviewer easily to the next question regarding
potential solutions.
THE BIG TRUCK COMPANY

Question 5: What specific steps would you take to achieve the savings
identified in Exhibit 2?

• Sample “Good” Answer: My first action would be to halt the practice of these sales
immediately to avoid further sales at a loss. I would then work very quickly with our
sales managers to structure new contracts so we can launch the business again. I
recognize this may be a key sell point for our salesman and do not want to leave
them without this tool. At the same time, I would begin an analysis of how affective
of a tool this business is for driving business. There may be a reason to be creating
contracts that drive a small loss if it allows us to take profitable new truck sale
volume from our competitors and creates greater profitability overall.

• Sample “Great” Answer: [Answer would be inclusive of the statement above].


After stopping the current unprofitable practice, understanding more appropriately
how to drive sales, and creating new procedures, I would then go back to the
current contracts and see how we can squeeze value out of them. We may be
able to renegotiate some of the contracts finding alternate solutions that are more
profitable for us. Perhaps we can break the contracts early. Perhaps we can
negotiate service packages in trade for buybacks at a higher profitability level to
the company. Maybe we can sell the contracts to a third party who thinks they
can get more value out of them than we can. Exploring as many options as
possible to increase profitability would be a worthwhile exercise.
THE BIG TRUCK COMPANY

Question 5 (cont’d):

Color Commentary: The interviewee should very deliberately talk about


things the company should do, preferably in a timeline (short, medium, and
long term) format. Thinking through not only what to do going forward, but
what to do about the current position is what truly makes an answer great.
The interviewer should be open to using the phrase “what else?” to push
the interviewer outside their comfort zone.
THE BIG TRUCK COMPANY

Recommendation
Question 6: It appears the potential for savings is there. The CEO would like
a specific recommendation as to what action he should take. Can you
provide a recommendation?

• Sample “Good” Answer: After extensive analysis of both the cost of manufacturing and
of the buy back program, I have identified significant savings in both areas. By modifying
the buy back program, we could potentially save $125 million, keeping in mind that these
obligations are unlikely to disappear entirely. As for the manufacturing, we have
identified an annual savings of $48 million, again under the caveat that some upfront
costs may be needed in order to realize the savings. I recommend you begin by
modifying your buy back program first and then looking more closely at your
manufacturing and suppliers in hopes of achieving the additional savings there as well.
When modifying the buy back program, you do want to be careful not to damage
relationships with your customers. However, the scale is large and needs to be addressed
regardless.

• Sample “Great” Answer: [Include the above recommendation, but add the following.]
Given this recommendation and the inherent risks, I would begin tomorrow with
communication to your sales team to begin implementing a solution to the buy back
program. Additionally, we should begin doing deep analysis on our manufacturing
operations in order to identify our inefficiencies and talking with our suppliers about how
to reduce costs. We should also look into the future to see if we can take some of the
savings for this line of trucks and apply it across the many product lines that we sell.
Additionally, given an impending downturn, we should also perform an analysis of the
effects of a changed sales program as it may affect the implementation.
THE BIG TRUCK COMPANY

Question 6 (cont’d):

Color Commentary: A good answer should have a clear, early


recommendation, evidence for the recommendation, risks inherent in the
changes, and clear next steps to be taken in the immediate future. Credit
should also be given for noticing that an improvement to operations
creates perpetual annual savings. The interviewee should be comfortable
asking for a moment to synthesize their thoughts and structure an answer
starting with the action, then reasons, then risks, and finally next steps.
THE BIG TRUCK COMPANY

Exhibits
THE BIG TRUCK COMPANY
CASE 18:
MACHINE COMPANY
Firm Style Interview Round
McKinsey 2
Case Question:
Our client is Machine Co. – a European specialty machine manufacturer. The company researches,
designs, and manufactures a variety of machines from underwater drills to semi-conductor chip
fabricating machines. The machines are developed based on specific requirement of a customer, as well
as brand products which are then marketed and sold to niche customers. One of Machine Co.’s brand
products is an injection molding machine (XG 43). This machine injects and molds plastic at high
temperatures to create specific plastic products. XG43 is renowned for fabricating high quality products
with a nine-sigma defect rate. The quality of products is unmatched by any other existing machine.
Although XG43 has strong and profitable market share in Europe, it is unable to sell any machines in the US.
Over the past few years, Machine Co. has made two unsuccessful attempts to penetrate the US market.
McKinsey and Co. has been retained by Machine Co. to evaluate whether a third attempt is warranted,
and if so, how can the attempt be made successful.

Clarifying Questions & Answers


Provide the following answers only if the interviewee asks the corresponding questions.
Question Answer

For this discussion, we are only going to focus on XG43’s potential in the US? Yes

Do we have any information as to why Machine Co.’s previous two attempts No, but let’s find
at US market were unsuccessful? (low hanging fruit) out
Success for entering a new market entry can be measured by market share Sure
and breakeven. Is Machine Co. using the same metrics?
MACHINE COMPANY

Framework / Structure
This is a market entry case. The analysis may include, but is not
limited to, the following areas:
• Understand XG43’s value proposition
• High quality was mentioned, but what else?
• Investigate other value propositions
• High efficiency/speed (No)
• Low power consumption (No)
• Low material waste ** (yes, asking or arriving at this is 20% of
the case), reduces waste by 10%
• Understand the US market
• Size of injection molding market (segment by low quality, high
quality, etc.)
• US competitors, and how XG43 compares with them
• Is the value proposition enough for them to switch
MACHINE COMPANY

Framework / Structure (cont’d)


• Executing the entry
• Brand awareness (none required. XG43 has received rave reviews in
industry journals
and has been endorsed by independent agencies.)
• Infrastructure
• Manufacturing location (in Europe, so none)
• Service (none exists in US, but 10 will be needed to deliver the
service that will convince users to switch)
• ** Getting this is 40% of the case. This is the reason why two
previous attempts were unsuccessful.
• When an injection molding machine breaks down, its needs to
be fixed right away, so a technician will have to be flown in the
same day to avoid costly loses
MACHINE COMPANY

• Economics/Breakeven
• Each Service center will cost $1 MM/year
• Current fuel injection molding machine in the US costs $150K
• Machine Co. would like price its machines at $450 K (which is
equivalent to European prices)
• Every year about 100 machines are ordered, only 10% are put to
high quality manufacturing
• Machines reduces waste by 10%, each machine wastes 1 tons of
product daily, Each lbs of molten plastic costs $2.50 cents.
• Breakeven for consumers in 2 years (** 40% of the case)
• Life of each XG43 is 10 years.
MACHINE COMPANY

Recommendation
My recommendation is a definite GO for US market entry
• Previous attempts have failed because of a lack of service
infrastructure in the US, which is key to service machines on same day
(needed to convince customers)
• Value Proposition
• For 10% of machine buyers, value prop is strong (high quality)
• For remaining 90%, reduction of waste will be enough to break
even on the XG43’s $300K price premium in two years.
• High probability of success
• Should capture almost all the high quality segment (10 machines
per year)
• Should capture significant portion of the remaining segment due
to waste reduction cost savings and high quality products
• Next Steps
• Hiring, training and retaining technical staff will be key
• Will need to develop a US based sales staff to convince customers

Das könnte Ihnen auch gefallen